Рассмотрите равенства умножить на 3 7: Рассмотрите равенства *** умножить на 3= ***7 *** умножить на 7= ***3

Содержание

Урок 67. деление на 3 — Математика — 2 класс

Математика 2 класс. Урок № 67

Деление на 3

Перечень вопросов, рассматриваемых в теме:

  1. Какое действие обратное умножению?
  2. Как найти неизвестный множитель?
  3. Как составить таблицу деления на 3 и таблицу, когда частное равно 3?
  4. Для чего необходимо знать деление на 3?

Глоссарий по теме:

Умножение – это сложение одинаковых слагаемых. Знак умножения — ‧, х.

Компоненты умножения: первый множитель, второй множитель.

Результат умножения – произведение.

Деление – действие обратное умножению.

Компоненты деления: делимое, делитель, частное.

Делимое – число, которое делят.

Делитель – число, на которое делят.

Частное – результат деления.

Обязательная литература и дополнительная литература:

  1. Моро М.И., Бантова М.А. и др. Математика 2 класс. Учебник для общеобразовательных организаций М.; Просвещение, 2017. – с. 92
  2. М. И. Моро, С. И. Волкова. Для тех, кто любит математику 2 класс. Учебное пособие для общеобразовательных организаций. М.; Просвещение,2018. – с. 57

Теоретический материал для самостоятельного изучения

Рассмотрите равенство. 3 • 5 = 15, где 3 – первый множитель, 5 – второй множитель, 15 – произведение.

Действие деление обратное действию умножения. Если произведение разделить на один из множителей, то получится второй множитель. Поэтому составим записи на деление.

15 : 3 = 5

15 : 5 = 3

Перед вами таблица умножения числа 3.

3 • 2 = 6

3 • 3 = 9

3 • 4 = 12

3 • 5 = 15

3 • 6 = 18

3 • 7 = 21

3 • 8 = 24

3 • 9 = 27

Пользуясь данной таблицей, можно легко составить таблицу, где делитель равен 3.

6 : 3 = 2

9 : 3 = 3

12 : 3 = 4

15 : 3 = 5

18 : 3 = 6

21 : 3 = 7

24 : 3 = 8

27 : 3 = 9

И таблицу, где частное равно 3.

6 : 2 = 3

9 : 3 = 3

12 : 4 = 3

15 : 5 = 3

18 : 6 = 3

21 : 7 = 3

24 : 8 = 3

27 : 9 = 3

Мы составили таблицу деления на число 3 и таблицу, когда в частном получается 3. Достаточно знать хотя бы один из предложенных столбиков таблицы, можно быстро найти значение выражений.

Для чего необходимо знать деление на 3? Знание помогает при решении задач. Например, такой.

У Димы в пакете 12 конфет. Пакет порвался, и мальчик решил их разложить в 3 кармана поровну. Сколько конфет в одном кармане?

12 : 3 = 4 (конф.)

В каждом кармане по 4 конфеты.

Вот еще одна задача.

Бабушка разлила 6 литров варенья в двухлитровые банки. Сколько банок с вареньем получилось у бабушки?

6 : 2 = 3 (б.)

У бабушки получилось 3 банки с вареньем.

Выполним несколько тренировочных заданий.

Рассмотрите рисунок. Составьте записи по рисунку.

5 • 3 = 15

15 : 5 = 3

15 : 3 = 5

Рассмотрите рисунок, составьте записи на умножение и деление.

Проверьте.

4 • 3 = 12 12 : 3 = 4 12 : 4 = 3

Решим задачу. Игрокам раздали 12 теннисных мячей, по 3 мяча каждому. Сколько игроков получили мячи?

Выполним рисунок.

Для решения выбираем действие деление, так как неизвестно количество игроков, получивших мячи.

Решение задачи:

12 : 3 = 4 (игрока).

Ответ: 4 игрока.

Решим еще одну задачу.

Раздали 12 теннисных мячей четырем игрокам поровну. Сколько мячей получил каждый игрок?

Выполним рисунок. Каждый раз будем брать по 4 мяча (по числу игроков0 и раздавать игрокам по 1 мячу до тех пор, пока не останется ни одного мяча.

Решение задачи:

12 : 4 = 3 (мяча)

Ответ: по 3 мяча.

Вывод:

Ответим на вопросы, поставленные в начале урока.

Чтобы найти неизвестный множитель, надо произведение разделить на известный множитель.

Действие деление обратное действию умножения.

Если произведение разделить на один из множителей, то получится второй множитель.

Зная таблицу умножения числа 3 и взаимосвязи между компонентами действия умножения, можно составить таблицу деления на 3 и таблицу, когда в частном число 3.

Знание таблицы деления на 3 помогает быстро выполнять вычисления и решать задачи.

Выполним несколько тренировочных заданий.

1. Выполните вычисления.

3 • 5 18 : 3 27 : 9

2 • 3 21 : 3 6 : 2

Пользуясь таблицей умножения числа 3 и таблицей деления, найдем значение выражений.

3 • 5 = 15 18 : 3 = 6 27 : 9 = 3

2 • 3 = 6 21 : 3 = 7 6 : 2 = 3

2. Закончите записи, чтобы получились верные равенства.

3 • 8 = 243 • 5 = □4 • □ = 12

24 : 3 = □ 15 : □ = 3 12 : 3 = □

24 : 8 = □ 15 : □ = 5 □ : 4 = 3

Получим верные равенства

3 • 8 = 243 • 5 = 154 • 3 = 12

24 : 3 = 8 15 : 5 = 3 12 : 3 = 4

24 : 8 = 3 15 : 3 = 5 12 : 4 = 3

3. Решите задачу.

За 3 часа работы трактор расходует 21 литр топлива. Сколько литров топлива расходует трактор за 1 час?

21 : 3 = 7 (л)

Ответ: 7 литров за 1 час.

Конспект урока по математике 2 класс по теме » Связь между компонентами и результатом умножения.

Конспект урока по математике, 2 класс

Тема: «Связь между компонентами и результатом умножения».

Цели: раскрыть связь между компонентами и результатом умножения; совершенствовать вычислительные навыки и навыки устного счёта; развивать внимание и логическое мышление.

Задачи урока: предметные – научить моделировать с помощью схематических рисунков действия умножения и деления; находить множители с помощью взаимосвязи умножения и деления; выполнять устные вычисления изученных видов в пределах 100; выполнять задания творческого и поискового характера; контролировать и оценивать свою работу и ее результат.

Ход урока.

  1. Организационный момент

  2. Актуализация знаний.

1).Фронтальная работа.

— Разделите выражения на две группы ( выражения записаны на доске. Учащиеся показывают знаки на карточках).

a … с + а n . 5 … n + n . 4

b … b – 5 38 – n … 68 – n

а . b … b . а с . 1 … с + 1

а + b … b + а а – а … b – b

k + k + k … k . 2

2).Работа над задачами ( решения показываются на индивидуальных наборных полотнах, ошибки сразу разбираются и анализируются ).

  • На одной тарелке 10 слив. Сколько слив на 2 таких тарелках?

  • Мама испекла 12 пирожков и разложила их на 2 тарелки поровну. Сколько пирожков на каждой тарелке?

  • У Светы было 8 яблок. Она угостила своих друзей, дав каждому по 2 яблока. Сколько друзей у Светы?

— Какая из этих задач лишняя?

— Сравните задачи на деление?

3.Самоопределение к деятельности.

На данном уроке мы рассмотрим тему «Взаимосвязь между компонентами и результатом умножения», которая поможет нам расширить представления об арифметических действиях. Мы поговорим о взаимосвязи между произведением и множителем, о том, как относятся друг к другу компоненты и результат умножения.

Посмотрите на следующее выражение:

7 ∙ 3

Давайте найдем его значение, а поможет нам в этом сложение. Мы помним, что умножение – это сложение одинаковых чисел. Внимательно посмотрите на выражение 7∙ 3.

Какое слагаемое будет повторяться?(Это слагаемое 7.)

Сколько раз будет повторяться число 7?( Число 7 будет повторяться 3 раза).

7 ∙ 3

7 + 7 + 7 = 21

Мы заменяли сложением предыдущее выражение. Значит, значение выражения 7 ∙ 3 будет 21.

7 ∙ 3 = 21

Давайте прочтем это выражение, используя термины: первый множитель, второй множитель и произведение.

Первый множитель – 7, второй множитель – 3, произведение – 21.

Давайте выясним, как из первого равенства получили второе равенство и третье равенство.

7 ∙ 3 = 21

21 : 7 = 3

21 : 3 = 7

Вы заметили, что произведение разделили на первый множитель и в результате получили второй множитель.

Во втором равенстве мы произведение разделили на второй множитель, а в результате получили первый множитель.

Взаимосвязь умножения и деления

Давайте посмотрим на три других равенства.

5 ∙ 2 = 10

10 : 5 = 2

10 : 2 = 5

Как они связаны между собой?

Вы заметили, что если произведение разделить на первый множитель, то в результате мы получим второй множитель. А если произведение разделить на второй множитель, то получим первый множитель.

Давайте сделаем вывод.

Если произведение двух чисел разделить на один из множителей, то в результате получится второй множитель.

Теперь давайте попробуем использовать эти знания для выполнения заданий. В этом нам поможет равенство:

4 ∙ 3 = 12

Давайте сравним значения выражений:

12 : 3 … 12 : 4

Мы помним, что если произведение разделить на один из множителей, то в результате получится второй.

12 : 3 = 4

Значит значение первого выражения – 4.

12 : 4 = 3

Значение второго выражения – 3.

4 больше, чем 3. Это значит, что значение первого выражения больше, чем значение второго.

12 : 3 12 : 4

4 3

Выполним аналогичное задание, но с другими числами. Посмотрите внимательно на выражение, в котором выполнено действие умножение.

6 ∙ 5 = 30 (первый множитель – 6, второй множитель – 5, произведение – 30)

30 : 6 … 30 : 5

Если произведение разделить на первый множитель, мы получим второй множитель. Это значит, что результат выражения 30 : 6 равняется 5.

Если произведение разделить на второй множитель, то в результате мы получим первый множитель.

30 : 5 = 6.

Сравним числа 5 и 6. 5 меньше 6. Это значит, что значение первого выражения меньше, чем значение второго.

30 : 6

5

Давайте решим задачу.

Условие задачи:

Бабушка принесла яблоки и разложила их на 2 тарелки, по 4 яблока на каждую. Сколько яблок принесла бабушка?

Давайте разложим яблоки на две тарелки так, как это сделала бабушка. Обозначим вопрос задачи. (Рис. 1).

Рис. 1. Яблоки и тарелки

Чтобы решить эту задачу, можно воспользоваться сложением. Но поскольку на тарелках одинаковое количество яблок, то сложение мы можем заменить умножением. Давайте составим выражение и найдем его результат. 4 яблока у нас повторяется 2 раза.

4 ∙ 2 = 8 (яблок)

Теперь давайте решим задачу, обратную данной. Как в каждой задаче, известное будет неизвестным. И наоборот. Посмотрите не нашу новую задачу.

Условие задачи:

Бабушка принесла 8 яблок и разложила их на 2 тарелки поровну. Сколько яблок на каждой тарелке? (Рис. 2).

Рис. 2. Иллюстрация к задаче

Для того чтобы решить эту задачу, мы воспользуемся решением прошлой задачи.

4 ∙ 2 = 8 (яблок)

Всего было 8 яблок, их разложили на две тарелки.

8 : 2

Для того чтобы найти результат, нам поможет решение прошлой задачи. Мы знаем, что если произведение разделить на один из множителей, то получим другой множитель.

8 : 2 = 4 (ябл.)

На этом уроке мы узнали, как связаны между собой компоненты и результат умножения.

Откройте учебник на стр. 72 и прочитайте задачи урока.

4. Работа по теме урока.

1). Выводы взаимосвязи множителей и произведения.

— Как математики называют результат умножения? ( Произведение.)

— А числа, которые умножаем? ( Множители.)

( Учитель закрепляет на доске карточку.)

М . М = П

— Запишите выражение: множители 2 и 3, произведение 6 и проиллюстрируйте его рисунком.

( 2 . 3 = 6 )

( Учащийся выписывает пример и рисунок на доске. )

— Прочитайте его с названием компонентов.

— Какие еще выражения можно составить с этими числами , используя этот рисунок? ( 3 . 2 = 6 6 : 2 = 3 6 : 3 = 2 )

— Что вы заметили? Попробуем сделать вывод.

— Прочитайте эти выражения, используя названия компонентов при умножении, начиная со слова «Если..» ( Если произведение разделить на первый множитель, то получится второй множитель. Если произведение разделить на второй множитель, то получится первый множитель.)

— Прочитаем правило в учебнике.

— Что вы можете сказать о наших рассуждениях?

( Учитель прикрепляет на доске карточки)

М = П : М М = П : М

2). Работа по учебнику.

— Рассмотрите рисунки на стр. 62 и равенства под ними.

— Прочитайте первое равенство, называя компоненты умножения.

— Как нашли первый множитель?

— Как нашли второй множитель

( Аналогично разбираются примеры второго столбика.)

3). Работа в группах по 4 человека.

№1 с 72

— Дополните каждый столбик выражением, используя эти же числа.

— Какое свойство вы использовали? ( Переместительное.)

5. Физкультминутка.

Елочки зеленые

На ветру качаются,

На ветру качаются,

Низко наклоняются.

Сколько елочек зеленых,

Столько сделайте наклонов.

Приседайте столько раз,

Сколько бабочек у нас.

Сколько беленьких кружков,

Столько сделайте прыжков.

6. Закрепление изученного материала.

Работа по учебнику.

№3 стр. 72.

— Прочитайте задачу.

— Решите ее.( Решение самостоятельное.)

— Обсуждение вариантов решения и объяснение ошибок самими учащимися.

( Варианты решения на доске пишут учащиеся)

— Составьте обратные задачи.

( Учащиеся предлагают свои варианты и объясняют, почему выбрали действие деление.)

№4 стр. 72.

— Решите задачу.

— Почему мы не можем ее решить? ( Нет данных.)

— Вставьте пропущенные числа и решите ее с помощью схемы.

( На доске рассматриваются разные способы решения задачи.)

№ 5 стр. 72.

7. Подведение итогов.

3 класс, часть 1 – 2 Консультация 3. Уроки 1 – 13.

3 класс, часть 1 – 2

Консультация 3. Уроки 1 – 13.

На уроках 1 – 5 систематизируются знания учащихся о единицах измерения длины и массы, вводятся новые единицы измерения массы: грамм, центнер, тонна, закрепляются соотношения между единицами измерения длины, массы, умение выражать значения величин в разных единицах измерения. Также повторяются и закрепляются нумерация и действия с многозначными числами, решение текстовых задач, уравнений, примеров на порядок действий, умножение чисел в столбик, измерение отрезков и построение отрезков данной длины, понятие объема прямоугольного параллелепипеда, отрабатываются вычислительные навыки.

На уроке 1 воспроизводится таблица, устанавливающая соотношение между единицами длины, с которой учащиеся уже встречались раньше:

Теперь область применения этой таблицы существенно расширяется. В 1, стр. 95 проговариваются все возможные соотношения между этими единицами. Например, устанавливается, что 1 км = 1000 м = 10 000 дм = 100 000 см = 1 000 000 мм и т. д. При этом надо вспомнить правило: при переходе к меньшим меркам выполняется умножение, а при переходе к большим меркам – деление. Соответствующие коэффициенты перехода (числа, на которые надо умножать или делить при переходе от одной единицы измерения к другой) записаны под дугами.

В 2–4, стр. 95 учащиеся используют установленные соотношения и аналогию с десятичной системой записи чисел для перевода длин из одних единиц измерения в другие. Решение примеров записывается в тетради в клетку и проговаривается вслух. Способ обоснования может быть различным – на основе установленного правила либо на основе аналогии с десятичной системой записи чисел, например:

а) 7 м = 700 см, так как в 1 метре 100 сантиметров, а 100 · 7 = 700,

или

7 м = 700 см, так как 7 метров – это 7 сотен сантиметров;

б) 16 000 мм = 1600 см, так как в 1 сантиметре 10 миллиметров, а

16 000 : 10 = 1600,

или

16 000 мм = 1600 см, так как в 16 000 содержится 1600 десятков;

в) 12 км 50 м = 12 050 м, так как в 1 километре 1000 метров, значит,

в 12 км – 12 000 м, да еще 50 м, всего получится 12 050 метров,

или

12 км 50 м = 12 050 м, так как 12 км 50 м – это 12 тысяч 50 метров.

Основным способом является первый, так как он универсальный и используется, например, и при преобразовании единиц времени, где соотношения между единицами не являются десятичными. Однако акцент на аналогию системы мер длины и массы с десятичной системой записи чисел не только поможет закрепить знание нумерации, но и покажет связь изучения чисел с практическими задачами. Каждый из учеников может выбрать тот способ обоснования, который ему удобен, а в классе должны звучать оба способа.

Перед выполнением заданий 5–6, стр. 96 надо повторить с учащимися правило о том, что величины можно сравнивать, складывать и вычитать только тогда, когда они выражены в одних и тех же единицах измерения. Поэтому для сравнения, сложения и вычитания величин в этих заданиях надо их сначала выразить в одинаковых мерках.

На уроке 2 в 1–2, стр. 98 учащиеся решают практические задачи, связанные с построением отрезков и измерением их длин. В 1 они устанавливают, что если точки A, B и C лежат на одной прямой, то длина AC равна сумме длин AB и BC, а если нет, то длина AC меньше суммы длин AB и . Другими словами, прямая линия, соединяющая две точки A и C, короче ломаной ABC. В 2 они строят планы земельных участков треугольной и четырехугольной формы и вычисляют их периметры. Таким образом, их внимание еще раз обращается на то, что числа возникли для решения практических задач, поэтому естественно, что соотношения между единицами измерения величин аналогичны принципу нумерации. Эта аналогия еще раз подчеркивается в 3, стр. 98. В заданиях 4–5, стр. 98 рассматриваются более сложные случаи перевода единиц длины.

На уроках 3–4 аналогичным образом рассматриваются единицы массы и соотношения между ними:

Правило перевода единиц и способы перевода остаются прежними, изменяются лишь названия единиц и переводные коэффициенты. Кроме того, рассматриваются виды гирь, которые обычно используются при взвешивании, и способы уравновешивания предметов на чашечных весах.

Хотим отметить, что при выполнении 10, стр. 99 следует обратить внимание на некоторые моменты. К настоящему времени дети уже знают, что одни и те же математические выражения могут описывать разнообразные жизненные ситуации. Так, выражение 2 + 3 может быть суммой игрушек, ручек, тракторов и еще чего угодно, в том числе «шклидулок». И от того, что мы не знаем, что такое «шклидулка», суть вычислений не изменится – мы все равно получим в ответе 5.

В задаче предлагается вымышленная ситуация – о шклидулках и бримазятах. Математическая структура задачи не представляет для учеников труда, но здесь они должны суметь перенести ее на абстрактное для них содержание и провести рассуждения во всей полноте.

– Чтобы ответить на первый вопрос задачи, можно сложить шклидулки, которые нашли бримазище и бримазенок. (Ищем целое.) Для этого сначала из 96 вычтем 64 и узнаем, сколько шклидулок нашел бримазенок. Чтобы узнать, во сколько раз больше шклидулок нашел бримазище, чем бримазенок, надо первое число разделить на второе.

1) 96 – 64 = 32 (ш.) – нашел бримазенок.

2) 96 + 32 = 128 (ш.).

3) 96 : 32 = 3 (раза).

Ответ: вместе они нашли 128 шклидулок, бримазище – в 3 раза больше бримазенка.

При выполнении 12, стр. 103 следует рассуждать так:

Р – 70 Г – 200 С – 40

И – 80 К – 5400 Б – 400

П – 50 О – 4800 Н – 100

СПРИНГБОК. Один из интереснейших видов газелей, обитающий в Южной Африке. Верхняя сторона тела – желто‑коричневая, нижняя сторона – белая, на границе проходит контрастная буровато‑черная полоса. Но самая замечательная особенность спрингбока – обширная продольная кожная складка на спине. Когда животное спокойно, складку не видно. Но, почувствовав опасность, спрингбок начинает подпрыгивать на месте, отталкиваясь одновременно всеми ногами, без видимых усилий, как резиновый мяч.

Прыжки спрингбока колоссальны: до 2 м в высоту. При этом края кожной складки расходятся, и выстилающий ее белый мех начинает ослепительно сверкать. Для всех обитателей саванны прыжки спрингбока служат сигналом опасности.

Спрингбок знаменит своими странствиями. К сожалению, говорить о них приходится лишь в прошедшем времени: они прекратились вместе с резким уменьшением численности спрингбока. Во время последнего крупного переселения спрингбоков в 1896 году животные плотной массой покрывали участок шириной около 25 км, а длина колонны составляла 220 км!

Во второй части учебника закрепляются нумерация, сложение и вычитание многозначных чисел, вводится умножение и деление многозначного числа на однозначное, рассматриваются некоторые преобразования на плоскости (параллельный перенос, симметрия), меры времени и календарь, на основе некоторых логических понятий (высказывание, истинное и ложное высказывание) уточняется понятие уравнения и рассматриваются новые их виды. Учащиеся знакомятся с понятиями переменной и выражения с переменной, учатся находить значения выражений с переменной, строить формулы зависимостей между величинами.

На уроках 6 – 9 у учащихся формируется умение умножать многозначные числа на однозначные и умножать круглые числа в случаях, сводящихся к умножению на однозначное число, учатся решать задачи на нахождение значений величин по их сумме и разности. Ученики повторяют и закрепляют нумерацию, сложение и вычитание многозначных чисел, решение текстовых задач, решение уравнений с комментированием по компонентам действий, сравнение выражений, действия с единицами длины и массы.

Простейшие случаи умножения многозначного числа на однозначное (27 · 5, 140 · 3 и т. д.) и их запись в столбик уже встречались учащимся. На данном этапе обучения они должны распространить известный им способ умножения в столбик на общий случай умножения многозначного числа на однозначное, и отработать его для сложных случаев. Работа ведется, как обычно, деятельностным методом.

На уроке 6 на этапе актуализации знаний с учащимися нужно вспомнить распределительное свойство умножения. Для этого можно рассмотреть с ними различные способы нахождения площади прямоугольников для случаев, когда прямоугольник разбит на 2 части и на 3 части:

По данным рисункам ставятся вопросы:

1) Чем похожи и чем отличаются эти задачи? (В первой задаче прямоугольник разбит на две части, а во второй – на три.)

2) Как называется первое равенство? (Правило умножения суммы на число, или распределительное свойство умножения.)

3) Можно ли распространить это правило на сумму трех слагаемых? (Из второго равенства следует, что да.)

4) Можно ли его распространить на сумму большего числа слагаемых? (Да, ведь прямоугольник можно разбить на большее число частей.)

Чтобы поставить проблему, учащимся можно сначала предложить решить в тетрадях в клетку следующие примеры и выявить в них закономерности:

Ученики могут заметить, что:

1) все примеры – на умножение;

2) первый множитель увеличивается, а второй не изменяется;

3) с увеличением первого множителя произведение увеличивается;

4) если первый множитель увеличивается в 10 раз, то и все произведение

увеличивается в 10 раз.

Затем учитель предлагает, воспользоваться тем же вычислительным приемом и решить пример

При решении примера, вероятно, возникнет затруднение: могут получиться разные ответы, кто‑то из детей не решит его и т. д. Возникшая проблемная ситуация и мотивирует поиск нового способа действий.

В случае, если с последним примером справятся все обучающиеся, можно попросить их обосновать решение. Главное – дети должны заметить, что для решения данного примера используется другой вычислительный прием. Этот признак отличия они должны проговорить вслух: в первых четырех примерах требуется умножить двузначное число на однозначное, а в последнем примере – трехзначное на однозначное.

После этого цель урока может быть сформулирована следующим образом: установить, как умножается любое многозначное число на однозначное. Если последний пример выполнят все ученики, то цель урока мотивируется необходимостью обосновать правомерность используемого приема.

Этап «открытия» нового знания начинается с выбора метода рассуждений. Рассмотренная в начале урока задача о вычислении площадей прямоугольников должна помочь учащимся вспомнить, что алгоритм умножения двузначного числа на однозначное был установлен на основе правила умножения суммы на число (распределительного свойства умножения), и сориентироваться на это свойство.

В 1, стр. 1 еще раз проговаривается формулировка правила умножения суммы на число и возможность его распространения на любое число слагаемых. Затем в 2 (а), стр. 1 данное число 576 разбивается на удобные слагаемые 500 + 70 + 6 и на основе этого правила выполняются преобразования:

Очевидно, что такая запись является слишком громоздкой, неудобной, – это учащиеся скажут сразу. Тогда ставится задача найти более короткий способ записи по аналогии с умножением на двузначное число. Если самостоятельно ученики затруднятся это сделать, можно предложить им проанализировать слагаемые суммы по рисунку 2 (б), стр. 1. Дети должны заметить, что при вычислении суммы сначала подсчитывается число единиц, затем число десятков и число сотен (нули при сложении результата не изменяют). И поскольку все эти числа всегда являются двузначными (значения табличных произведений), то удобнее число единиц следующего разряда, которое «запоминается», писать вверху над соответствующим разрядом первого множителя, как при умножении двузначных чисел. Подвести учащихся к этому выводу можно следующей последовательностью вопросов:

1) Как получили слагаемые суммы? (6 единиц умножили на 9, потом 7 десятков умножили на 9, а потом 5 сотен умножили на 9.)

2) Всегда ли во втором слагаемом на конце будет нуль? Почему? (Всегда, так как считаем число десятков.)

3) Всегда ли в третьем слагаемом на конце 2 нуля? Почему? (Всегда, так как считаем число сотен.)

4) Почему во втором столбике нули зачеркнуты? (Они не изменяют значение суммы.)

5) Может ли число единиц, десятков или сотен «заходить» не на один следующий разряд, а на 2 или 3 разряда? (Нет, перемножаем однозначные числа, поэтому в произведении не может быть больше двух знаков.)

6) Сравните запись умножения во втором и третьем столбике – какая из записей удобнее? (В третьем столбике.)

7) Догадайтесь, как она получается из предыдущей? (Сначала умножаем единицы: 6 · 9 = 54, 4 единицы пишем, а 5 десятков запоминаем – записываем над числом десятков первого множителя. Потом умножаем десятки: 7 · 9 = 63, 63 + 5 = 68, 8 десятков пишем, а 6 сотен запоминаем. А потом умножаем сотни: 5 · 9 = 45, 45 + 6 = 51, записываем 51 сотню. – «Открытие».)

Пишу: множитель 9 под разрядом единиц множителя 576.

Умножаю единицы: 6 · 9 = 54 ед., пишу 4 в разряде единиц,

а 5 д. запоминаю.

Умножаю десятки: 7 · 9 = 63 д., 63 + 5 = 68 д., пишу 8 в разряде

десятков, а 6 с. запоминаю.

Умножаю сотни: 5 · 9 = 45 с., 45 + 6 = 51 с., пишу 1 в разряде

сотен, а 5 – в разряде тысяч.

Ответ: 5184.

В завершение учитель спрашивает у детей, изменятся ли рассуждения при умножении на однозначное число четырехзначного, пятизначного, шестизначного и т. д. числа. Как правило, дети легко распространяют полученный вывод на любое многозначное число. Тогда в тетради в клетку надо записать, решить и прокомментировать (с возможной помощью учителя) более сложный случай умножения, например, 5 · 20 156. Внимание детей обращается на порядок множителей и на то, что в данном случае также удобно писать однозначный множитель под разрядом единиц многозначного множителя.

Если у учащихся все же возникнет сомнение в правомерности распространения полученного вывода на случай умножения любого многозначного числа на однозначное, то можно рассмотреть аналогичным образом умножение четырехзначного числа на однозначное или предложить учащимся сделать это дома самостоятельно.

Примеры для этапа первичного закрепления подбираются в зависимости от уровня подготовленности класса. Можно, например, решить с подробным комментированием в громкой речи 3 (а), стр. 1, а для этапа самоконтроля использовать 3 (б), стр. 1. После выполнения самостоятельной работы ученики сопоставляют свое решение с образцом, предъявленным учителем, и убеждаются в том, что новый вычислительный прием ими освоен. Напомним, что при изучении нового материала первостепенное значение имеет создание ситуации успеха для каждого ребенка. Возможные ошибки должны здесь же исправляться, а материалы дорабатываться индивидуально, пока остальные учащиеся класса решают задачи на повторение.

На этапе повторения новое знание включается в систему знаний, а также решаются задания, обеспечивающие непрерывность развития содержательно‑методических линий курса. Так, на рассматриваемом уроке умножение многозначного числа на однозначное встречается при решении текстовых задач 4–5, стр. 2, в уравнении 6, стр. 2 и при работе с буквенными выражениями в 7, стр. 2. Далее в задании 8, стр. 2 повторяется правило порядка действий в выражениях и отрабатываются вычислительные навыки. В 9, стр. 2 повторяются действия с многозначными числами, в 10–11, стр. 2 – понятия равенства и пересечения множеств, которые связываются с рисованием геометрических фигур и перебором вариантов, а в 12, стр. 2 предлагается логическая задача. Учитель на уроке введения нового знания выбирает для оставшихся 5–10 минут урока из этих заданий те, в которых учащиеся его класса испытывают больше затруднений.

Сделать этот выбор более осознанным и обоснованным позволяют «Электронные приложения к учебникам».

С другой стороны, методическим приемом, который позволяет существенно увеличить число решенных в классе примеров без перегрузки детей, является решение задач по выбору учащихся. Так, например, на данном уроке учитель может предложить учащимся на этапе повторения решить по выбору одно из заданий 5–9, стр. 2. Учащиеся в течение 3–4 минут решают по одному выбранному ими заданию, а затем проговаривают их решение в течение следующих 5 минут. Таким образом, все задания воспроизведены в памяти детей, т. е. цель повторения достигнута. При этом в классе создается атмосфера психологической комфортности, так как каждый ребенок решает задание, которое он выбрал сам, а значит, то, которое ему больше понравилось. Задачи по выбору можно предлагать и для домашней работы.

При подведении итога урока учитель обсуждает с учениками вопросы:

– Что нового узнали? (Научились умножать любое многозначное число на однозначное.)

– Какое математическое свойство для этого использовали? (Распределительное свойство умножения.)

– Кто уже чувствует себя уверенно в решении новых примеров?

– Что повторили? Что больше всего понравилось?

– Кто сегодня нам помогал на уроке?

– Как оцениваете свою работу?

Для домашней работы можно предложить учащимся придумать и решить свой пример на умножение многозначного числа на однозначное, решить задачу 4, стр. 2 и по желанию – одно из заданий 10–12, стр. 2. Таким образом, обязательное задание не займет у обучающихся больше 10–15 мин самостоятельной работы. При таком подходе исключена перегрузка детей, каждому из них обеспечивается возможность успешного усвоения необходимого минимума, и в то же время каждому предоставляется возможность обучения на высоком уровне за счет активного включения в деятельность на уроке и решения дополнительных развивающих заданий.

На уроках 7–8 рассматриваются более сложные случаи умножения многозначного числа на однозначное и случаи умножения круглых чисел, сводящиеся к ним. Так, в 1, стр. 6 учащиеся распространяют на множество многозначных чисел изученное ранее правило: чтобы умножить круглые числа, надо выполнить умножение, не глядя на нули, а потом к полученному произведению приписать столько нулей, сколько в обоих множителях вместе. На основании этого правила при записи умножения круглых чисел в столбик для удобства вычислений нули мысленно отбрасываются и полученное однозначное число записывается в разряде единиц многозначного множителя:

На последующих уроках умножение многозначного числа на однозначное отрабатывается в основном в процессе выполнения проверки примеров на деление.

На уроке 8 рассматривается новый тип задач – задачи на нахождение величин по их сумме и разности. На основе предметных действий с моделями полосками ученики догадываются, что при вычитании из суммы двух чисел их разности получается удвоенное меньшее число, а при сложении суммы и разности – удвоенное большее число. Поэтому решить задачу, например, 1, стр. 8 можно двумя способами:

Для этапа первичного закрепления предназначены задания 3–4, стр. 8–9, а для этапа самостоятельной работы с самопроверкой в классе – 2, стр. 8. Дома можно предложить им придумать и решить свои задачи на нахождение величин по их сумме и разности.

На всех данных и последующих уроках особое внимание уделяется комментированию решения уравнений по компонентам действий ( 6, стр. 2; 6, стр. 4; 6, стр. 9; 7, стр. 18; 5, стр. 20; 4, стр. 25 и т. д.). Это связано с подготовкой детей к изучению темы «Уравнения» на уроке 27 данной части учебника. К этому времени обучающиеся должны не только уметь на автоматизированном уровне верно находить неизвестные компоненты действий, но и комментировать решение по образцу, приведенному на стр. 77 учебника.

На уроках 9 – 12 формируется умение делить многозначные числа на однозначные и делить круглые числа, сводящиеся к делению на однозначное число, умение делать проверку деления умножением, а также повторяются и закрепляются нумерация, сложение и вычитание многозначных чисел, умножение многозначного числа на однозначное, решение текстовых задач. Учащиеся решают уравнения с комментированием по компонентам действий, повторяют понятие периметра треугольника, понятие числового луча, действия с единицами длины и массы, читают и записывают выражения.

При изучении внетабличного деления в пределах 100 учащиеся знакомились с правилом деления суммы на число. Сейчас это правило используется для построения алгоритма деления многозначного числа на однозначное. В итоге обсуждения учащиеся должны выявить и осмыслить основную идею, основной принцип деления многозначных чисел: сначала делится более крупная счетная единица, затем остаток дробится и делится следующая по величине счетная единица и так далее до конца. Новый материал вводится в обучение деятельностным методом.

На уроке 9 на этапе актуализации знаний с учащимися нужно вспомнить взаимосвязь между умножением и делением (a : b = c b · c = a, b 0), алгоритм деления с остатком и правило деления суммы на число, распространив его, как и в предыдущем случае, на сумму трех и более слагаемых.

На этапе постановки проблемы детям можно предложить в течение 2–3 минут в тетрадях в клетку самостоятельно решить примеры «по частям», т. е. используя правило деления суммы на число, и выявить в них закономерности:

Учащиеся могут заметить, что:

1) все примеры – на деление;

2) делимое увеличивается, а делитель не изменяется;

3) с увеличением делимого частное увеличивается;

4) если делимое увеличивается в 10 раз, то и частное увеличивается в 10 раз.

При решении последнего примера обычно возникает затруднение, которое мотивирует поиск нового способа действий (если и последний пример выполнят все ученики, можно попросить их найти лишний пример).

Далее учитель подводит учащихся к выявлению существенного для данного урока признака отличия последнего примера от предыдущих: первые четыре примера сводятся к делению двузначного числа на однозначное, а в последнем примере – деление трехзначного числа на однозначное. Этот признак отличия учащиеся должны проговорить вслух.

Таким образом, ставится цель урока установить, как делится многозначное число на однозначное. (Если затруднений в решении последнего примера у обучающихся не возникнет, слово установить заменяется словом обосновать – ведь подобные примеры в классе ранее не рассматривались.)

На этапе «открытия» нового знания детям вначале предоставляется возможность выбрать метод рассуждений. Задания, рассмотренные в начале урока, должны сориентировать их на выбор правила деления суммы на число, распространенного на случай нескольких слагаемых. Для подбора слагаемых для вычисления частного 536 : 4 можно использовать графическую модель. Учитель рисует ее на доске, а учащиеся – в тетради:

Рассматривая ее, ученики должны догадаться, что для нахождения частного вначале надо разделить сотни (коробки), затем оставшуюся сотню перевести в десятки и делить все имеющиеся десятки (пачки) и, наконец, оставшийся десяток раздробить в единицы (штуки) и делить единицы. В менее подготовленных классах поиск решения целесообразно сопровождать не только графическим моделированием, но и предметным – работой с конкретными коробками, пачками и единицами предметов.

Получившиеся группы обводятся овалами – это «удобные слагаемые»:

Из приведенных рассуждений следует, что каждый получил 1 сотню, 3 десятка и 4 штуки, или 134 штуки предметов. На математическом языке проведенные рассуждения можно записать так:

536 : 4 = (400 + 120 + 16) : 4 = 400 : 4 + 120 : 4 + 16 : 4 = 100 + 30 + 4 = 134.

Эта цепочка преобразований записывается в тетрадь, и еще раз проговаривается полученный вывод: чтобы разделить многозначное число на однозначное, можно делимое разбить на сумму «удобных» слагаемых и делить «по частям», то есть по правилу деления суммы на число.

Применение этого способа действий весьма ограничено, но проведенные рассуждения помогут учащимся в дальнейшем осмыслить общий принцип деления многозначных чисел. Для перехода к делению углом надо показать им неудобство построенного способа действий, предложив, например, найти частное 11 768 : 4.

Понятно, что попытки найти «удобные» слагаемые вряд ли закончатся успехом, и тогда можно попросить детей еще раз вернуться к рисунку:

– Рассмотрите, с каких единиц мы начинали деление – с мелких или с крупных? (С крупных.)

– Конечно, ведь удобнее сначала раздать более крупные счетные единицы – коробки. Но вот у нас 1 коробка осталась, что нам пришлось сделать? (Достать пачки и делить уже пачки.)

– Правильно, нам пришлось раздробить сотни в десятки. А когда и десятки у нас закончились, что мы сделали? (Стали делить единицы.)

– Кто теперь догадается, как можно делить любое многозначное число, не подбирая слагаемые? (Делить сначала самые крупные счетные единицы, затем остаток дробить и делить более мелкие единицы.)

На доске в процессе беседы учитель кратко записывает суть выполняемых преобразований:

1) 5 с. : 4 = 1 с. (ост. 1 с.)

2) 13 д. : 4 = 3 д. (ост. 1 д.)

3) 16 ед. : 4 = 4 ед. Итак, 536 : 4 = 134.

Аналогично записывается решение примера 11 768 : 4, предложенного учителем:

1) 11 т. : 4 = 2 т. (ост. 3 т.)

2) 37 с. : 4 = 9 с. (ост. 1 с.)

3) 16 д. : 4 = 4 д.

4) 8 ед. : 4 = 2 ед. Итак, 11 768 : 4 = 2942.

Таким образом, поставленная проблема решена: найден общий способ деления многозначного числа на однозначное. Он заключается в делении с остатком возможно более крупных счетных единиц и последовательном переходе к делению более мелких счетных единиц. Однако остается проблема записи деления. На вопрос учителя: «Удобная ли запись деления?» – ответ всегда одинаковый: неудобная, громоздкая. Тогда можно предложить учащимся попробовать придумать свою запись, более короткую и удобную. Для этой цели лучше использовать первый пример – 536 : 4.

Только после того как дети предложат свои версии, следует показать им «свернутый» способ записи приведенных рассуждений – уголком, и прокомментировать его:

Проверку деления удобно делать умножением на основании взаимосвязи:

Так, для проверки выполненного деления можно число 2942 умножить на 4.

Учитель обращает внимание учащихся на то, что при комментировании примеров надо вначале указать первое неполное делимое, потом определить число цифр в частном, а затем рассказать, как находятся цифры в каждом разряде частного. При этом надо постоянно помнить о том, что на каждом шаге мы фактически выполняем деление с остатком, и поэтому получаемые остатки должны быть меньше делителя. Проверку решения удобно делать умножением.

Алгоритм письменного деления фиксируется с помощью блок-схемы:

Проблема разрешена.

Для проведения этапа первичного закрепления можно использовать задания 3–6, стр. 11–12, которые решаются с проговариванием в громкой речи. В 3 учащиеся находят частное всеми тремя рассмотренными способами. В 4 внимание детей еще раз фиксируется на том, что остаток от деления всегда должен быть меньше делителя, проговариваются основные этапы деления многозначного числа на однозначное, выделенные в рамке на стр. 11. Примеры 5–6 записываются в тетради в клетку и решаются по выбору. Здесь возможно комментирование в паре, в группе, создание игровых ситуаций. Достаточно, если каждый ребенок решит 2–3 примера. Параллельно проговаривается способ проверки деления умножением, зависимость между компонентами деления.

Задание 2, стр. 10 целесообразно использовать на этапе самостоятельной работы с самопроверкой в классе. Оно менее сложное, чем примеры, решенные на предыдущем этапе урока, и содержит наглядную опору, которая поможет обучающимся лучше представить каждый этап деления.

На этапе повторения по выбору можно решить задания 7 (а), стр. 12 и 9 (а), стр. 12.

При подведении итога урока обсуждаются вопросы:

– Что нового узнали? (Научились делить многозначное число на однозначное, записывать деление «углом».)

– Какой прием используется для устного деления? (Деление «по частям».)

– С каких единиц начинаем письменное деление? (С самых крупных.) А потом? (Делим по очереди более мелкие единицы.)

– Кто сегодня нам хорошо помогал?

– Кто доволен своей работой?

– Что повторили? Что больше всего понравилось?

В домашней работе можно предложить учащимся самостоятельно составить и решить пример на деление трехзначного числа на однозначное, построить его графическую модель и выполнить деление тремя способами по аналогии с тем, как это сделано в учебнике. Кроме того, решить по собственному выбору одно из заданий 7 (б), 9 (б), стр. 12. В качестве дополнительного задания, которое выполняется по желанию, – одно из заданий 8, 10, стр. 12.

На последующих уроках рассматриваются более сложные случаи деления: делимое содержит большее число цифр (урок 10), в частном получаются нули в середине и на конце (уроки 11–13).

Чтобы переварить знания, надо поглощать их с аппетитом.

(А. Франц)

Желаем Вам удачи и творческих успехов!

Мы вместе, значит, у нас все получится!

Урок по математике на тему «Умножение числа 8 на однозначное число»

дата: 25.01.2018

урок: математика

тема: умножение числа 8 на однозначные числа

Цели: составить столбик умножения числа 8 на однозначные числа; закреплять знание таблицы умножения на 2, 3, 4, 5, 6, 7; совершенствовать умение составлять равенства; продолжить формирование умения решать задачи; повторить правило нахождения периметра многоугольника; развивать умение анализировать и сравнивать.

Ход урока

I. Организационный момент.

II. Устный счет.

1. Выполните действия и вставьте числа в «окошки».

III. Работа по теме урока.

– Сегодня на уроке составим восьмой столбик «Таблицы умножения».

1. Задание 1.

– Рассмотрите схему вычислений. Запишите суммы в виде произведений и вычислите их значения. Для вычисления следующего значения используйте предыдущее.

8 + 8 + 8 + 8 + 8 + 8 + 8 + 8 = 8 · 8 = 64

8 + 8 + 8 + 8 + 8 + 8 + 8 + 8 + 8 = 8 · 9 = 72

– При помощи полученных результатов завершите заполнение восьмого столбика «Таблицы умножения».

– Сколько в этом столбике однозначных значений произведений? (1.)

– Сколько двузначных значений? (8.)

– Сколько среди двузначных значений таких, в составе которых 1 десяток? (1.) 2 десятка? (1.) 3 десятка? (1.) 4 десятка? (1.) 5 десятков? (1.) 6 десятков? (1.) 7 десятков? (1.)

– Какие строчки этого столбика вы уже запомнили?

2. Задание 2.

Учащиеся работают по учебнику (с. 159, раздел «Сделай сам»).

– Найдите в незаполненном столбике «Таблицы умножения» произведение, в котором второй множитель равен числу 8.

– Какую строчку оно занимает в этом столбике? (Восьмую строчку.)

– Подчеркните в «Таблице умножения» эту строчку.

– Вычислите и запишите в «Таблицу умножения» значение этого произведения.

– Какое правило вы использовали? (Закон перестановки множителей.)

3. Задание 3.

– Среди данных выражений найдите те, которые имеют одинаковые значения.

– Составьте из них равенства и запишите их.

8 + 8 = 8 · 2

8 + 8 + 8 + 8 = 8 + 8 · 3

8 + 8 + 8 + 8 + 8 + 8 + 8 + 8 = 8 · 8

4. «Таблица умножения» в стихах.

Учащиеся читают отрывок из стихотворения «Таблица умножения» Марины Казариной.

Вот цифра ВОСЕМЬ на подходе!

Чтоб даром время не терять,

Начнём-ка, братцы, умножать!

Восьмерку на один умножит

Подводный житель осьминог,

Ходить по суше он не может,

Хоть и имеет ВОСЕМЬ ног!

А восемь на два – знайте, братцы,

Решенье верное – ШЕСТНАДЦАТЬ!

А восемь на три – не забыли?

Ответ «в часах» – ДВАДЦАТЬ ЧЕТЫРЕ!

Умножим восемь на четыре,

Здесь только ТРИДЦАТЬ ДВА, друзья,

Хоть в лукоморье говорили

Про тридцать три богатыря!

Умножим восемь на пятёрку –

Здесь СОРОК, вариантов нет!

А вот подсказка-поговорка:

«За сорок бед – один ответ!»

Восьмёрочку на шесть умножим –

Выходит СОРОК ВОСЕМЬ здесь!

Ну а на семь помножив, сможем

Мы получить – ПЯТЬДЕСЯТ ШЕСТЬ!

На восемь восемь научились

Мы без ошибок умножать,

И ровно ШЕСТЬДЕСЯТ ЧЕТЫРЕ

Должны в ответе указать!

На девять восемь умножаем.

Вот результат: СЕМЬДЕСЯТ ДВА!

На десять восемь – отвечаем:

Здесь ВОСЕМЬДЕСЯТ, господа!

Ура! Восьмёрку одолели!

Ещё рывок – и мы у цели!

Физкультминутка

5. Задание 4.

– Дополните условие и сформулируйте требование так, чтобы решением получившейся задачи стало произведение 8 · 3.

Задача. В каждой из трех коробок лежало по 3 кг моркови. Сколько всего килограммов моркови?

6. Задание 5.

– Сравните данные выражения. Сколько действий содержит каждое выражение?

– Какое действие выполняется первым? (Умножение.)

– Разделите данные выражения на группы, используя схемы:  ·  + ,  ·  – ,  +  · .

 ·  + 

 ·  – 

 +  · 

8 · 5 + 8 = 48

8 · 7 + 8 = 64

8 · 8 + 8 = 72

8 · 6 – 8 = 40

8 · 8 – 8 = 56

8 · 9 – 8 = 64

8 + 8 · 8 = 72

8 + 8 · 0 = 8

7. Задание 6.

– Каждая сторона пятиугольника имеет длину 8 см. Чему равен периметр этого пятиугольника?

Р = 8 · 5 = 40 (см)

– Что обозначает первый множитель?

– Что обозначает второй множитель?

8. Задание 7.

– Каждая сторона восьмиугольника имеет длину 5 см. Чему равен периметр этого восьмиугольника?

Р = 5 · 8 = 40 (см)

– Что обозначает первый множитель?

– Что обозначает второй множитель?

9. Самостоятельная работа.

1. Вставь числа в «окошки», чтобы получились верные равенства:

8 · 6 = 8 + 8 + 8 +  +  8 · 8 = 8 ·  – 8

8 · 7 = 8 · 6 +  8 · 3 = 8 + 8 + 

8 · 9 = 9 · 9 –  8 · 5 = 8 ·  + 8

5 · 8 =  · 5 8 · 5 = 8 ·  – 8

2. Запиши выражения в виде произведения двух чисел и найди их значения.

5 · 6 + 11 7 · 7 – 13

9 · 4 + 23 6 · 6 – 25

54 + 3 · 7 46 – 4 · 6

3. Какому рисунку соответствует каждое выражение и что оно обозначает?

– Проверь себя, пользуясь линейкой и вычислениями.

– Начерти ломаную линию, которой соответствует выражение: .

IV. Итог урока.

– Что нового узнали на уроке?

Домашнее задание: составить и решить задачу по выражению 6 · 8.

Использование образовательной технологии «Школа 2100» в обучении математике младших школьников (стр. 12 из 15)

— Что интересного в записанных числах? (Записаны разными цве­тами; все “красные” числа — четные, а “синие” — нечетные.)

— Какое число лишнее? (10 — круглое, а остальные нет; 10 — дву­значное, а остальные однозначные; 5 — повторяется два раза, а осталь­ные — по одному.)

— Закрою число 10. Есть ли лишнее среди остальных чисел? (3 — у него нет пары до 10, а у остальных есть.)

— Найдите сумму всех “красных” чисел и запишите ее в красном квадрате. (30.)

— Найдите сумму всех “синих” чисел и запишите ее в синем квад­рате. (23.)

— На сколько 30 больше, чем 23? (На 7.)

— На сколько 23 меньше, чем 30? (Тоже на 7.)

— Каким действием искали? (Вычитанием.)

2.2. Задания на развитие памяти и речи. Актуализация знаний.

а) —Повторите по порядку слова, которые я назову: слагаемое, сла­гаемое, сумма, уменьшаемое, вычитаемое, разность. (Дети пытаются воспроизвести порядок слов.)

— Компоненты каких действий назвали? (Сложение и вычитание.)

— С каким новым действием мы познакомились? (Умножение.)

— Назовите компоненты умножения. (Множитель, множитель, про­изведение.)

— Что обозначает первый множитель? (Равные слагаемые в сумме.)

— Что обозначает второй множитель? (Число таких слагаемых.)

Запишите определение умножения.

б) —Рассмотрите записи. Какое задание будете выполнять?

12 + 12 + 12 + 12 + 12

33 + 33 + 33 + 33

а + а + а

(Заменить сумму произведением.)

Что получится? (В первом выражении 5 слагаемых, каждый из которых равен 12, поэтому оно равно

12 • 5. Аналогично — 33 • 4, а • 3)

в) — Назовите обратную операцию. (Заменить произведение суммой.)

— Замените произведение суммой в выражениях: 99 — 2. 8 • 4. Ь • 3. (99 + 99, 8 + 8 + 8 + 8, b+b+b).

г) На доске записаны равенства:

81+81=81–2

21• 3 = 21+22 + 23

44 + 44 + 44 + 44 = 44 + 4

17 + 17-17 + 17-17 = 17 • 5

Учитель рядом с каждым равенством помещает картинки соот­ветственно цыпленка, слоненка, лягушонка и мышонка.

— Зверюшки лесной школы выполняли задание. Правильно ли они его выполнили?

Дети устанавливают, что слоненок, лягушонок и мышонок ошиб­лись, объясняют, в чем их ошибки.

д) — Сравните выражения:

8 – 5… 5 – 8 34 – 9… 31 • 2

5 • 6… 3 • 6 а – 3… а • 2 + а

(8 • 5 = 5 • 8, так как от перестановки слагаемых сумма не изменя­ется; 5 • 6 > 3 • 6, так как слева и справа по 6 слагаемых, но слева слага­емые больше; 34 • 9 > 31 — 2. так как слева слагаемых больше и сами слагаемые больше; а • 3 = а • 2 + а, так как слева и справа по 3 слагае­мых, равных а.)

— Какое свойство умножения использовали в первом примере? (Переместительное.)

2.3. Постановка проблемы. Целеполагание.

Рассмотрите картинку. Верны ли равенства? Почему? (Верны, так как сумма 5 + 5 + 5= 15. потом в сумме становится на одно слагае­мое 5 больше, и сумма увеличивается на 5.)

5 • 3 = 15 5 • 5 = 25

5 • 4 = 20 5 • 6 = 30

— Продолжите эту закономерность направо. (5 • 7 = 35; 5 • 8 = 40…)

— Продолжите ее теперь налево. (5 • 2 = 10; 5 • 1=5; 5 • 0 = 0.)

— А что означает выражение 5 • 1? 5 • 0? (? Проблема!) Итог обсуждения:

— В нашем примере было бы удобно считать, что 5 • 1 = 5, а 5 • 0 = 0. Однако выражения 5 • 1 и 5 • 0 не имеют смысла. Мы можем условиться считать эти равенства верными. Но для этого надо проверить, не нарушим ли мы переместительное свойство умножения. Итак, цель нашего урока — установить, сможем ли мы считать равенства 5 1 = 5 и 5 0 = 0 верными? — Проблема урока!

3. “Открытие” детьми нового знания.

1) № 1, стр. 80.

а) — Выполните действия: 1 • 7, 1 • 4, 1 • 5.

Дети решают примеры с комментированием в учебнике-тетради:

1 • 7 = 1 + 1 + 1 + 1 + 1 + 1 + 1 = 7

1 • 4 = 1 + 1 + 1 + 1 = 4

1 • 5 = 1 + 1 + 1 + 1 +1 = 5

— Сделайте вывод: 1 • а — ? (1 • а = а.) Учитель выставляет карточку: 1 • а = а

б) — Имеют ли смысл выражения 7 • 1, 4 • 1, 5 • 1? Почему? (Нет, так как в сумме не может быть одно слагаемое.)

— Чему они должны быть равны, чтобы не нарушалось переместительное свойство умножения? (7 • 1 тоже должно быть равно 7, поэтому 7 • 1 = 7.)

Аналогично рассматриваются 4 • 1 = 4; 5 • 1 = 5.

— Сделайте вывод: а • 1 = ? (а • 1 = а.)

Выставляется карточка: а • 1 = а. Учитель накладывает первую карточку на вторую: а • 1 = 1 • а = а.

— Совпадает наш вывод с тем, что у нас получилось на числовом луче? (Да.)

— Переведите это равенство на русский язык. (При умножении числа на 1 или 1 на число получается то же самое число.)

— Молодцы! Итак, будем считать:

а • 1 = 1 • а = а.

2) Аналогично исследуется случай умножения с 0 в № 4, стр. 80. Вывод — приумножении числа на 0 или 0 на число получается нуль:

а • 0 = 0 • а = 0.

— Сравните оба равенства: что вам напоминают 0 и 1?

Дети высказывают свои версии. Можно обратить их внимание на те образы, которые приведены в учебнике: 1 — “зеркальце”, 0 — “страш­ный зверь” или “шапка-невидимка”.

Молодцы! Итак, при умножении на 1 получается то же самое число (1 — “зеркальце”), а при умножении на 0 получается 0 (0 — “шапка-невидимка”).

4. Физкультминутка.

5. Первичное закрепление.

На доске записаны примеры:

23 • 1 = 0 • 925 = 364 • 1 =

1 • 89= 156 • 0 = 0 • 1 =

Дети решают их в тетради с проговариванием в громкой речи полученных правил, например:

3 • 1 = 3, так как при умножении числа на 1 получается то же самое число (1 — “зеркальце”), и т.д.

2) № 1, стр. 80.

а) 145 • х = 145; б) х • 437 = 437.

При умножении 145 на неизвестное число получилось 145. Значит, умножали на 1• х= 1. И т.д.

3) № 6, стр. 81.

a) 8 • x = 0; б) х • 1= 0.

— При умножении 8 на неизвестное число получился 0. Значит, умножали на 0 • х = 0. И т.д.

6. Самостоятельная работа с проверкой в классе.

1) № 2, стр. 80.

1 • 729 = 956 • 1 = 11 =

№5, стр. 81.

0 • 294 = 876 • 0 = 0 • 0 = 1 • 0 =

Дети самостоятельно решают записанные примеры. Затем по го­товому образцу проверяют свои ответы с проговариванием в громкой речи, отмечают правильно решенные примеры плюсом, исправляют допущенные ошибки. Те, кто допустил ошибки, получают аналогич­ное задание на карточке и дорабатывают индивидуально с учителем, пока класс решает задачи на повторение.

7. Задачи на повторение.

а) — Мы сегодня приглашены в гости, а к кому? Вы узнаете, рас­шифровав запись:

[Р] (18 + 2) — 8 [О] (42+ 9)+ 8

[А] 14 — (4 + 3) [Н] 48 + 26 — 26

[Ф] 9 + (8 — 1) [Т] 15 + 23 — 15

У каждого ученика — карточка с заданием. Дети самостоятельно выполняют вычисления и расшифровывают запись:

— К кому же мы приглашены в гости? (К Фортрану.)

б) — Профессор Фортран — знаток компьютеров. Но дело в том, что у нас нет адреса. Кот Икс — лучший ученик профессора Фортрана — оставил для нас программу (Вывешивается плакат такой, как на стра­нице 56, М-2, ч. 1.) Отправляемся в путь по программе Икса, К какому домику пришли?

Один ученик по плакату на доске, а остальные — в учебниках выполняют программу и находят дом Фортрана.

в) — Нас встречает профессор Фортран со своими учениками. Его лучшая ученица — гусеница — приготовила для вас задание: “Я задума­ла число, вычла из него 7, прибавила 15, потом прибавила 4 и полу­чила 45. Какое число я задумала?”

-7 +15 +4

Обратные операции надо делать в обратном порядке: 45-4-15 + 7 = 31.

г) Игра-соревнование.

Асам профессор Фортран предложил нам поиграть в игру “Вычислительные машины”.

Таблица в тетрадях у учеников. Они самостоятельно выполняют вычисления и заполняют таблицу. Выигрывают первые 5 человек, которые справляются с заданием правильно.

8. Итог урока.

— Все ли сделали на уроке, что планировали?

— С какими новыми правилами познакомились?

Конспект урока математики по теме «Умножение числа 9, деление на 9» (3 класс).

Урок математики в 3­м классе по теме «Умножение числа 9, деление на 9» 
с использованием ИКТ
Дергачева О.А. учитель начальных классов
Цели урока:




Составить таблицы умножения девяти и на девять, рассмотреть соответствующие случаи 
деления.
Совершенствовать вычислительные навыки и умение работать с именованными числами
Развивать наблюдательность, логическое мышление и умение рассуждать.
Воспитывать аккуратность и трудолюбие.
Оборудование:


учебник «Математика. 3 класс I часть», Давыдов В.В., Горбов С.Ф. и др.
презентация .
ХОД УРОКА
1. Орг. момент 
­ Ну­ка посмотри, дружок,
  Ты готов начать урок?
  Все ль на месте?
  Все ль в порядке?
  Книжки, ручки и тетрадки?
  Все ли правильно сидят?
  И внимательно глядят?
2.Сообщение целей и задач урока. 
     ­ Прочитайте высказывание. Как его понимаете? Кому оно принадлежит? Кто это? 
­ Сегодня на уроке мы узнаем много нового и интересного. Будем решать примеры, задачи, 
работать с именованными числами.
­ Но не будем забывать, что без точного счета, не сдвинется с места любая работа! 
3. Устный счет.
– Начнем наш урок с веселого счета (слайд 2): перед вами цифрогусеница. Как быстрее 
посчитать сумму всех чисел, из которых она состоит? 9 х 8 + 6 х 3 + 1 + 8 = 72 + 18 + 9 = 99
– Что можете сказать об этом числе? (Оно двузначное, записано с помощью одной и той же 
цифры  «9», в этом числе 9 десятков и 9 единиц, это наибольшее двузначное число)
– Какое число получится, если слева уберем одну девятку? (9)
– Какое число получится, если к 99 справа припишем одну девятку? (999)
4.Работа в тетрадях.
­  Открываем тетради, записываем сегодняшнее число, классная работа .
– Сегодня на чистописании запишем цифру 9 числами:  9  99  999, повторяя, таким образом, до 
конца строки.
– Посмотрите на следующее задание.
­ Перед вами два столбика произведений. Рассмотрите их внимательно, подумайте, можно ли 
составить равенства?
7 х 6                        4 х 6
3 х 8                        4 х 4
2 х 9                        9 х 2
5 х 8                        6 х 7
7 х 4                        8 х 5
9 х 8                        4 х 7
6 х 4                        6 х 3
5 х 9
8 х 9
– На основании,  какого свойства умножения вы составляли эти равенства? (От перестановки 
множителей, произведение не меняется)
Это свойство нам сегодня пригодиться на уроке.
– Эти  столбики непростые. В них зашифрована тема нашего урока. Чтобы её расшифровать, 
надо найти значения произведений каждого столбика и с помощью ключа записать слова.
40, 72, 42, 28, 18, 24                                  28, 16, 72, 18, 40, 24, 45, 42 
Л   Ц    Т   И   Б   А                                    Е    М  Я    Н   Ж  У   И   О
42, 24, 18, 40, 28, 72, 24                            24, 16, 18, 42, 40, 28, 18, 45, 72
Т   А    Б   Л   И   Ц   А                              У   М   Н   О   Ж   Е   Н   И   Я
– Что у вас получилось?  (Таблица умножения)
– Какую цифру писали на чистописании? 
5. Значит тема нашего урока
 
6. Работа над новой темой.
­ А вы знаете, ребята, что 9­ка – это магическое число?
   1) У «9­ти» не будет пути, говорили в древности;
   2) В сказках действия часто происходят в « тридевятом царстве».
 : таблица умножения с числом 9. ­ Может и таблица умножения с числом 9, тоже обладает « магическими» особенностями? 
Проверим?
– Перед вами таблица умножения девяти:
9 х 2 =
9 х 3 =
9 х 4 =
9 х 5 =
9 х 6 =
9 х 7 =
9 х 8 =
9 х 9 =
­ Чтобы было проще нам считать,
   Мы Пифагорову таблицу
  В стихах решили написать.
  Но для начала по порядку
  Беремся умножать 9­ку.
( дети читают стихи и делают записи в тетрадях)
Умножить 9 на 2  просто,
А чтоб не забывать ответ,
Запомни: твой « гражданский» возраст
Начнётся в 18 лет!   
                                                 « Девятка на три», вслух считаем,
                                                  Здесь 27­ решенье есть!
                                                   А на 4 умножаем –
                                                   Получим ровно 36!  
  Совсем не сложно научиться
  На 5 девятку умножать!
  Должно в итоге получиться
  Произведенье 45!  
                                                                  А чтоб на 6 умножить 9,
                                                                   Нам ничего не нужно делать!
                                                                   Мы это с вами проходили,
                                                                  В ответе ­ 54!      
А вот и умница Мальвина
Прилежно учит Буратино,
И говорит ему: « Смотри,
Девятью семь – 63!   
                                                                  Девятью восемь – вот задача, Давай, работай, голова!
                                                                  Но нас не подвела удача,
                                                                  Даем ответ – 72!      
На 9 девять умножаем,
Ответ в таблице проверяем,
А равен судя по всему,
Он 81!             
       
– Какое свойство умножения помогло  нам найти значения этих произведений? (От 
перестановки множителей, произведение не меняется)
– Какой новый случай появился?  (9 х 9)
– Таким образом, из всей таблицы необходимо запомнить всего один случай,
т. к. все предыдущие были уже изучены ранее.
– Возможен ли здесь случай перестановки множителей?  (Нет, т. к. множители одинаковые)
– При изучении умножения, мы всегда рассматриваем соответствующие случаи деления. 
Составьте выражение на деление.  Каким правилом воспользуемся? (Если произведение двух 
чисел разделить на одно из них, то получится другое число –  81 : 9 = 9)
– Сколько случаев получится? Почему?  (Одинаковые множители)
 7. Физминутка. 
– Что происходит с результатами во всей таблице?
(Каждое следующее число увеличивается на 9)
­ А теперь, рассмотрите внимательно результаты таблицы ещё раз. Как образуются десятки в 
каждом ответе? ( Увеличиваются на 1) ( слайд…)
­ А как происходит образование единиц? ( уменьшаются на 1) ( слайд…)
­ Т. О. мы убедились, что 9 – это особенное число.
8. Работа с учебником. Закрепление.
– А теперь откроем учебники на стр. 60 № 210
­ Прочитайте задание. Что нужно сделать? Применим полученные знания.
( От каждого ряда выходят ученики)
9 * 8 =                  9 * 6 =                       9 * 7 =
.. : 9 = 8                …: 9 = 6                   … :9 =  7
9 * 9 =                   9 * 4 =                     9 * 5 =
… : 9 = 9             …: 9 = 4                   … : 9 = 5
­ Какие знания пригодились? Что знаете о делении и умножении?
Стр. 61 № 211                
­ Что интересного в этом номере? Прочитайте.
( 1 ст – девочки, 2 ст – мальчики. К доске выходит мальчик и девочка) 9 м7 дм – 5м 2 дм = 45 дм                            8 дм 5 см – 4 дм 3 см = 42см
9 м 7 дм – 5 м 9 дм = 38 дм                          8 дм 5 см – 4 дм 7 см = 38 см
 ­ Что нужно уметь? Что нужно знать?
­ Где  и когда в жизни нам могут пригодиться  эти знания? ( пошив одежды, стройка, выполнение
чертежей)
­ В строительстве домов это необходимо?
Задача. 
 В деревне построили 3 ряда новых домов по 9 в каждом. А в посёлке 9 рядов по 8 в 
каждом.  На сколько больше домов построили в поселке, чем в деревне?
­ О чем идёт речь в задаче? Где строили дома? Как строили? Что известно о деревне? Как 
строили в поселке?
­ Рассмотрите схему.
­ Можно ли сразу ответить? Почему? Что неизвестно? Как найти? Что узнаем вторым 
действием?
­ Зная теперь кол­во  новых домов и в деревне и в поселке, можно сравнить и узнать разницу?
­ Запишите самостоятельно. 
­ Какой ответ получился? Как находили?
­ Сформулируйте ответ. Для этого вернемся к вопросу.
­ Какая задача? Почему?
9. Итог урока.
­ Чему учились на уроке?  Что нового узнали? Что было интересно?
10. Домашнее  задание  стр. 62 № 216
11. Рефлексия  
  ­ Кто считает, что все понял и ему было легко на уроке, поднимите  зелёный  смайлик.
­ Кто считает, что не совсем еще владеет полученными знаниями, есть над чем поработать, 
поднимите жёлтый.
­ Кто совсем ничего не понял и ему было очень трудно, покажите розовый.
­ Что необходимо делать, чтобы не было ошибок и всё получалось? ( трудиться)

Три уравнения найти неизвестное слагаемое. Общие сведения об уравнениях. Когда корней нет

Урок 80-81. Тема: «Решение уравнений»

Цели:
учить решать уравнения с неизвестным слагаемым; повторить соотношение единиц длины; закреплять навыки вы-числений в столбик; развивать умения рассуждать и логически мыслить.

Планируемые результаты:

учащиеся научатся решать урав-нения на нахождение неизвестного слагаемого; выполнять пись-менные вычисления, используя изученные приемы; понимать причины успеха/неуспеха учебной деятельности.

Ход урока

I

. Организационный момент

II

. Актуализациязнаний

Математический диктант

1. На сколько 67 меньше 89? (На 22.)

2. Из 7 десятков вычесть 4 десятка. (30.)

3. Увеличить 23 на 32. (55.)

4. Какое число я уменьшила на 27 и получила 23? (50.)

5. На сколько нужно увеличить 43, чтобы получилось 70? (На 27.)

6. Из суммы чисел 9 и 6 вычесть 10. (5.)

7. Какое число нужно вычесть из 64, чтобы получилось 37? (27.)

8. К какому числу прибавили 0 и получили 44? (44.)

9. К 21 прибавить разность чисел 14 и 6. (29.)
10. Сумма чисел 33, 16,4 и 27. (80.)

(Проверка.Самооценка.)

III

. Самоопределение к деятельности

Составьте еще три примера, используя данный пример. 6 + 4=10

(Учитель записывает примеры на доске.) 4 + 6=10 10-4 = 6 10-6 = 4

Какое правило вы применили при составлении примера насложение? (От перестановки слагаемых сумма не меня-ется.)

Какое правило вы применили при составлении примера на вычитание? (Если из суммы вычесть одно слагаемое, то по-лучится другое слагаемое.)


Чтобы узнать тему урока, разгадайте кроссворд.

1. Они бывают числовые и буквенные. (Выражения.)

2.
Числа, которые складывают, называют. (Слагаемые.)

3. Число, из которого вычитают. (Уменьшаемое.)

4. Математический знак вычитания. (Минус.)

5. Равенство, которое содержит неизвестное число. (Уравнение.)

6. Сумма длин сторон фигуры. (Периметр.)

7. Выражение со знаком «плюс». (Сумма.)

8. Запись, в которой есть знак «равно». (Равенство.)

9. Наименьшее двузначное число. (Десять.)
10. Латинская буква. (Икс.)

Что получилось в выделенной строке? (Решение уравнений.)

Тема урока: «Решение уравнений с неизвестным слагае-мым». Какие задачи мы поставим перед собой?

IV

. Работа по теме урока

1. Работа по учебнику

Рассмотрите фишки домино на с. 7 учебника и примеры, записанные рядом. Как получены примеры на вычитание? Каким правилом воспользовались при их составлении? За-кончите вывод. (Чтобы найти неизвестное слагаемое, надо из суммы вычесть известное слагаемое.)

1 (с. 7).
(Устное выполнение.)

2

(с. 7).
(Коллективное выполнение с подробным объяснением.)

2. Самостоятельное решение уравнений

Вариант 1 Вариант 2

х + 45 = 92 75+х = 81

26+х = 50 х + 22 = 70

(Два ученика записывают решение на откидной доске. Про-верка. Самооценка.)

Решение:

х + 45 = 92 75 + х = 81

х = 92-45 х = 81-75

х =
47 х
= 6

26+х=50 х + 22 = 70

х =
50 – 26 х =
70 — 22

3. Работа по учебнику

3(с. 7).
(Устное выполнение.)

4 (с.

7).
(Самостоятельное выполнение.Тем, кто испытывает затрудне-ния, учитель дает карточку-помощницу с программой решения.) 1) Сколько стаканов малины собрала сестра?

2) Сколько стаканов малины собрали вместе?(Проверка.Самооценка.)

V

. Физкультминутка

Я иду, и ты идешь — раз, два, три.{Шаги на месте.)

Я пою, и ты поешь — раз, два, три.(Хлопки в ладоши.)

Мы идем и поем — раз, два, три.(Прыжки на месте.)

Очень дружно мы живем — раз, два, три.(Шаги на месте.)

VI

. Закрепление изученного материала

Работа по учебнику
№ 1 (с. 14).

Какие единицы длины вы знаете?

Сколько миллиметров в 1 см? (Самостоятельное выполнение.Проверка.)Решение:

5
см 3
мм = 53
мм

3
см 8
мм = 38 мм
№2 (с. 14).

(Самостоятельное выполнение.Проверка.)

1) Решение:

АВ= 3 см 5
мм, CD

= 5 см 5 мм;

5 см 5 мм — 3 см 5 мм = 2 см.

Ответ:
длина отрезка CD

на 2 см больше длины отрезка АВ.

2) Решение: ЕКМО
= 2 см + 4 см + 1 см 5 мм = 7 см 5 мм. №3(с. 14).

(Самостоятельное выполнение. Проверка. Самооценка.)

Решение:

2
см = 20
мм

4
см 2
мм > 40
мм 30
мм = 3
см

4
см 5
мм см

VII

. Рефлексия

(«Проверь себя» (учебник, с. 7). Самостоятельное выполне-ние. Проверка.)

Решение:
15+х = 35 х = 35-15 х = 20

VIII

. Подведение итогов урока

Какой вид уравнений вспомнили сегодня?

Как найти неизвестное слагаемое?

Кому нужна помощь?

Домашнее задание:
Рабочая тетрадь: № 10, 11 (с. 6).

Цели обучения
— решать уравнения способом подбора и на основе связи сложения и вычитания.

Цели урока

Все учащиеся смогут:

находить корень уравнения методом подбора

Большинство учащихся смогут:

уметь записывать и решать простые уравнения на нахождение неизвестного слагаемого

Некоторые учащиеся смогут:

с опорой на рисунок составлять и решать самостоятельно уравнения.

Предыдущие знания:
понимание системы чисел в пределах 100; умение проводить сравнения и использовать язык сравнения.

Ход урока

Создание коллаборативной среды

(психологические минутки)

Прозвенел звонок веселый.
Вы начать урок готовы?
Будем слушать, рассуждать,
И друг другу помогать!

Объединение в группы

Цель:
объединение учащихся на группы повышает познавательный интерес к уроку, сплоченность к работе в группе.
Повторение правила работы в группах

Актуализация жизненного опыта

Стратегия «Мозговой штурм» Использование толстых и тонких вопрос.
— Что такое уравнение? (Равенство с неизвестным называют уравнением)
— Как в уравнении обозначается неизвестное?
— Что значит решить уравнение? (Значит найти неизвестное)
— Назовите компоненты сложения?

Оценивание: Три хлопка

Стартер «Просмотр видеоролика» (развивающий мультик)
Метод «Стоп-кадр!»

Целеполагания на урок

— Вы догадались, чем мы будем сегодня на уроке заниматься?
— Что нам поможет достигнуть целей урока (узнать новое, научиться решать такие математические записи) (свой опыт, учитель, учебник)
Дети формулируют цель урока, я обобщаю.
— Сегодня на уроке вы узнаете как решать уравнения с неизвестными слагаемым

Исследование. Работа по учебнику.

Цель:
Исследовать материал учебника с. 46

Задание 1. Игра по учебнику «Вагончики в туннеле»

Работа в группе. Стратегия «Подумай, обсуди, поделись». Межпредметная связь обучение грамоте (слушание и говорение)

Игра «Вагончики в туннеле»

Сколько вагонов в туннеле?
6 + х = 18 и 2 + х = 14.
Ответ: 12 вагонов.

Дескриптор:

— составляет по рисунку уравнение
— находит значение буквы методом подбора.
— делает вывод(формулируют правило)

Обратная связь «Светофор»
Здесь я использую моделирования уравнения с целью
формирования умение решать уравнения с неизвестным слагаемым.

Задание 2. Работа в паре. «Помоги герою»

Игра «Помоги герою»

Для работы в паре я использую совместное обучение, которое передает знания и навыки между учащимися.
Самооценивание по дескриптору: «Большой палец»

Динамическая пауза. Музыкальная физминутка.

Задание 3. Работа в группе. «Подумай-найди пару, поделись!»

Дескрипторы:

— работает вся группа;
— составляет и решает самостоятельно уравнения с опорой на рисунок;
— делает вывод (формулируют правило).

Обратная связь «Колесо»
Применение (учитель — наблюдает, помогает, проверяет, ученик — решает вопросы, демонстрирует знания)

Взаимопроверка по слайдам
Здесь я использую работу в группе для улучшения процесса усвоения информации.

Задание 4. Игра в паре «Кубик» (попробуй)

Работа в группе: «Подумай-найди пару, поделись!»

Дескриптор:

— подставляет выпавшее число
— решает самостоятельно уравнение.

Здесь я использую активный метод в игровой форме который приводит к более глубокому пониманию решения уравнения с неизвестным слагаемым.
Оценивание по дескрипторам «Светофор»

Задание 5. Индивидуальное задание

Дифференцированные задания.
Задания выбраны для учеников с разными уровнями знаний.

Дескриптор:

  1. находит корень уравнения по числовому лучу;
  2. находит с помощью математических цифр и знаков корень уравнения;
  3. составляет по картинке составляет уравнение.

Самооценивание «Светофор» (проверка по эталону).

— Молодцы вы справились с этим заданием!
Здесь я использую дифференцированный подход для индивидуальных потребностей в обучении для каждого учащегося.

Итог урока. Рефлексия «Метод «Интервью»

— Над чем сегодня на уроке мы работали?
— Как найти неизвестное слагаемое?
— Чем является неизвестное слагаемое? (Частью)
— Достигли ли поставленной цели?
— Что будут делать те ребята, которые испытывали трудности при работе с уравнениями? (Высказывания учащихся)

Цель:
учитель узнает поняли ли ученики тему урока и свои просчеты, чтобы устранить на следующем уроке. (высказывание учащихся) (здесь я использую удовлетворительнее потребности учеников)
Взаимооценивание «2 звезды, 1 пожелание»

Рефлексия «Лесенка успеха» (дети размещают смайлики)

— Я могу решить уравнение с неизвестным слагаемым.
— Я могу научить другого …
— Я затрудняюсь в …
— Я ничего не понял …

Цель:
самооценивания своих достижений за урок.

Admin

Чтобы скачать материал или !

Долгий путь наработки навыков решения уравнений
начинается с решения самых первых и относительно простых уравнений. Под такими уравнениями мы подразумеваем уравнения, в левой части которых находится сумма, разность, произведение или частное двух чисел, одно из которых неизвестно, а в правой части стоит число. То есть, эти уравнения содержат неизвестное слагаемое, уменьшаемое, вычитаемое, множитель, делимое или делитель. О решении таких уравнений и пойдет речь в этой статье.

Здесь мы приведем правила, позволяющие находить неизвестное слагаемое, множитель и т.п. Причем будем сразу рассматривать применение этих правил на практике, решая характерные уравнения.

Навигация по странице.

Итак, подставляем в исходное уравнение 3+x=8
вместо x
число 5
, получаем 3+5=8
– это равенство верное, следовательно, мы правильно нашли неизвестное слагаемое. Если бы при проверке мы получили неверное числовое равенство, то это указало бы нам на то, что мы неверно решили уравнение. Основными причинами этого могут быть либо применение не того правила, которое нужно, либо вычислительные ошибки.

Как найти неизвестное уменьшаемое, вычитаемое?

Связь между сложением и вычитанием чисел, про которую мы уже упоминали в предыдущем пункте, позволяет получить правило нахождения неизвестного уменьшаемого через известное вычитаемое и разность, а также правило нахождения неизвестного вычитаемого через известное уменьшаемое и разность. Будем формулировать их по очереди, и сразу приводить решение соответствующих уравнений.

Чтобы найти неизвестное уменьшаемое, надо к разности прибавить вычитаемое.

Для примера рассмотрим уравнение x−2=5
. Оно содержит неизвестное уменьшаемое. Приведенное правило нам указывает, что для его отыскания мы должны к известной разности 5
прибавить известное вычитаемое 2
, имеем 5+2=7
. Таким образом, искомое уменьшаемое равно семи.

Если опустить пояснения, то решение записывается так:
x−2=5
,
x=5+2
,
x=7
.

Для самоконтроля выполним проверку. Подставляем в исходное уравнение найденное уменьшаемое, при этом получаем числовое равенство 7−2=5
. Оно верное, поэтому, можно быть уверенным, что мы верно определили значение неизвестного уменьшаемого.

Можно переходить к нахождению неизвестного вычитаемого. Оно находится с помощью сложения по следующему правилу: чтобы найти неизвестное вычитаемое, надо из уменьшаемого вычесть разность
.

Решим уравнение вида 9−x=4
с помощью записанного правила. В этом уравнении неизвестным является вычитаемое. Чтобы его найти, нам надо от известного уменьшаемого 9
отнять известную разность 4
, имеем 9−4=5
. Таким образом, искомое вычитаемое равно пяти.

Приведем краткий вариант решения этого уравнения:
9−x=4
,
x=9−4
,
x=5
.

Остается лишь проверить правильность найденного вычитаемого. Сделаем проверку, для чего подставим в исходное уравнение вместо x
найденное значение 5
, при этом получаем числовое равенство 9−5=4
. Оно верное, поэтому найденное нами значение вычитаемого правильное.

И прежде чем переходить к следующему правилу заметим, что в 6 классе рассматривается правило решения уравнений, которое позволяет выполнять перенос любого слагаемого из одной части уравнения в другую с противоположным знаком. Так вот все рассмотренные выше правила нахождения неизвестного слагаемого, уменьшаемого и вычитаемого с ним полностью согласованы.

Чтобы найти неизвестный множитель, надо…

Давайте взглянем на уравнения x·3=12
и 2·y=6
. В них неизвестное число является множителем в левой части, а произведение и второй множитель известны. Для нахождения неизвестного множителя можно использовать такое правило: чтобы найти неизвестный множитель, надо произведение разделить на известный множитель
.

В основе этого правила лежит то, что делению чисел мы придали смысл, обратный смыслу умножения. То есть, между умножением и делением существует связь: из равенства a·b=c
, в котором a≠0
и b≠0
следует, что c:a=b
и c:b=c
, и обратно.

Для примера найдем неизвестный множитель уравнения x·3=12
. Согласно правилу нам надо разделить известное произведение 12
на известный множитель 3
. Проведем : 12:3=4
. Таким образом, неизвестный множитель равен 4
.

Кратко решение уравнения записывается в виде последовательности равенств:
x·3=12
,
x=12:3
,
x=4
.

Желательно еще сделать проверку результата: подставляем в исходное уравнение вместо буквы найденное значение, получаем 4·3=12
– верное числовое равенство, поэтому мы верно нашли значение неизвестного множителя.

И еще один момент: действуя по изученному правилу, мы фактически выполняем деление обеих частей уравнения на отличный от нуля известный множитель. В 6 классе будет сказано, что обе части уравнения можно умножать и делить на одно и то же отличное от нуля число, это не влияет на корни уравнения.

Как найти неизвестное делимое, делитель?

В рамках нашей темы осталось разобраться, как найти неизвестное делимое при известном делителе и частном, а также как найти неизвестный делитель при известном делимом и частном. Ответить на эти вопросы позволяет уже упомянутая в предыдущем пункте связь между умножением и делением.

Чтобы найти неизвестное делимое, надо частное умножить на делитель.

Рассмотрим его применение на примере. Решим уравнение x:5=9
. Чтобы найти неизвестное делимое этого уравнения надо согласно правилу умножить известное частное 9
на известный делитель 5
, то есть, выполняем умножение натуральных чисел: 9·5=45
. Таким образом, искомое делимое равно 45
.

Покажем краткую запись решения:
x:5=9
,
x=9·5
,
x=45
.

Проверка подтверждает, что значение неизвестного делимого найдено верно. Действительно, при подстановке в исходное уравнение вместо переменной x
числа 45
оно обращается в верное числовое равенство 45:5=9
.

Заметим, что разобранное правило можно трактовать как умножение обеих частей уравнения на известный делитель. Такое преобразование не влияет на корни уравнения.

Переходим к правилу нахождения неизвестного делителя: чтобы найти неизвестный делитель, надо делимое разделить на частное
.

Рассмотрим пример. Найдем неизвестный делитель из уравнения 18:x=3
. Для этого нам нужно известное делимое 18
разделить на известное частное 3
, имеем 18:3=6
. Таким образом, искомый делитель равен шести.

Решение можно оформить и так:
18:x=3
,
x=18:3
,
x=6
.

Проверим этот результат для надежности: 18:6=3
– верное числовое равенство, следовательно, корень уравнения найден верно.

Понятно, что данное правило можно применять только тогда, когда частное отлично от нуля, чтобы не столкнуться с делением на нуль. Когда частное равно нулю, то возможны два случая. Если при этом делимое равно нулю, то есть, уравнение имеет вид 0:x=0
, то этому уравнению удовлетворяет любое отличное от нуля значение делителя. Иными словами, корнями такого уравнения являются любые числа, не равные нулю. Если же при равном нулю частном делимое отлично от нуля, то ни при каких значениях делителя исходное уравнение не обращается в верное числовое равенство, то есть, уравнение не имеет корней. Для иллюстрации приведем уравнение 5:x=0
, оно не имеет решений.

Совместное использование правил

Последовательное применение правил нахождения неизвестного слагаемого, уменьшаемого, вычитаемого, множителя, делимого и делителя позволяет решать и уравнения с единственной переменной более сложного вида. Разберемся с этим на примере.

Рассмотрим уравнение 3·x+1=7
. Сначала мы можем найти неизвестное слагаемое 3·x
, для этого надо от суммы 7
отнять известное слагаемое 1
, получаем 3·x=7−1
и дальше 3·x=6
. Теперь осталось найти неизвестный множитель, разделив произведение 6
на известный множитель 3
, имеем x=6:3
, откуда x=2
. Так найден корень исходного уравнения.

Для закрепления материала приведем краткое решение еще одного уравнения (2·x−7):3−5=2
.
(2·x−7):3−5=2
,
(2·x−7):3=2+5
,
(2·x−7):3=7
,
2·x−7=7·3
,
2·x−7=21
,
2·x=21+7
,
2·x=28
,
x=28:2
,
x=14
.

Список литературы.

  • Математика.
    . 4 класс. Учеб. для общеобразоват. учреждений. В 2 ч. Ч. 1 / [М. И. Моро, М. А. Бантова, Г. В. Бельтюкова и др.].- 8-е изд. — М.: Просвещение, 2011. — 112 с.: ил. — (Школа России). — ISBN 978-5-09-023769-7.
  • Математика
    : учеб. для 5 кл. общеобразоват. учреждений / Н. Я. Виленкин, В. И. Жохов, А. С. Чесноков, С. И. Шварцбурд. — 21-е изд., стер. — М.: Мнемозина, 2007. — 280 с.: ил. ISBN 5-346-00699-0.

§ 1 Как найти неизвестное слагаемое

Как найти корень уравнения, если неизвестно одно из слагаемых? В этом уроке рассмотрим метод решения уравнений на основе связи между слагаемыми и значением суммы.

Давайте решим такую задачу.

На клумбе росло 6 красных тюльпанов и 3 желтых. Сколько всего тюльпанов росло на клумбе? Запишем решение. Итак, росло 6 красных и 3 желтых тюльпана, следовательно, мы можем записать выражение 6+3, выполнив сложение, получим результат — на клумбе росло 9 тюльпанов.

Запишем решение. Итак, росло 6 красных и 3 желтых тюльпана, следовательно, мы можем записать выражение 6+3, выполнив сложение, получим результат — на клумбе росло 9 тюльпанов. 6 + 3 = 9.

Давайте изменим условие задачи. На клумбе росло 9 тюльпанов, 6 сорвали. Сколько тюльпанов осталось?

Чтобы узнать, сколько тюльпанов осталось на клумбе, необходимо из общего количества тюльпанов 9 вычесть сорванные цветы, их 6.

Произведем вычисления: 9-6 получим результат 3. На клумбе осталось 3 тюльпана.

Снова преобразуем эту задачу. Росло 9 тюльпанов, 3 сорвали. Сколько тюльпанов осталось?

Решение будет выглядеть так: из общего количества тюльпанов 9 необходимо вычесть сорванные цветы, их 3. Осталось 6 тюльпанов.

Давайте внимательно рассмотрим равенства и постараемся выяснить, каким образом они связаны между собой.

Как можно заметить, в этих равенствах записаны одни и те же числа и взаимообратные действия: сложение и вычитание.

Вернемся к решению первой задачи и рассмотрим выражение 6 + 3 = 9.

Давайте вспомним, как называются числа при сложении:

6 — это первое слагаемое

3 — второе слагаемое

9 — значение суммы

А теперь подумаем, как мы получили разности 9 — 6 = 3 и 9 — 3 = 6?

В равенстве 9 — 6 = 3 из значения суммы9 вычли первое слагаемое6, получили второе слагаемое3.

В равенстве 9 — 3 = 6 из значения суммы9 вычли второе слагаемое3, получили первое слагаемое6.

Следовательно, если из значения суммы вычесть первое слагаемое, то получится второе слагаемое, а если из значения суммы вычесть второе слагаемое, то получится первое слагаемое.

Сформулируем общее правило:

Чтобы найти неизвестное слагаемое, нужно из значения суммы вычесть известное слагаемое.

§ 2 Примеры решения уравнений с неизвестным слагаемым

Давайте рассмотрим уравнения с неизвестными слагаемыми и попробуем с помощью этого правила найти корни.

Решим уравнение Х + 5 = 7.

В этом уравнении неизвестно первое слагаемое. Чтобы его найти, воспользуемся правилом: чтобы найти неизвестное первое слагаемое X, необходимо из значения суммы 7 вычесть второе слагаемое 5.

Значит, Х = 7 — 5,

найдем разность 7 — 5 = 2 , Х = 2.

Проверим, правильно ли мы нашли корень уравнения. Для осуществления проверки необходимо подставить в уравнение вместо Х число 2:

7 = 7 — получили верное равенство. Делаем вывод: число 2 является корнем уравнения Х+5=7.

Решим еще одно уравнение 8 + У =17.

В этом уравнении неизвестно второе слагаемое.

Чтобы его найти, необходимо из значения суммы 17 вычесть первое слагаемое 8.

Сделаем проверку: подставим вместо У число 9. Получим:

17 = 17 — получили верное равенство.

Следовательно, число 9 является корнем уравнения 8 + У = 17.

Итак, на уроке мы познакомились с методом решения уравнений на основе связи между слагаемыми и значением суммы. Чтобы найти неизвестное слагаемое, нужно из значения суммы вычесть известное слагаемое.

Список использованной литературы:

  1. И.И. Аргинская, Е.И. Ивановская, С.Н. Кормишина. Математика: Учебник для 2 класса: В 2ч. — Самара: Издательство «Учебная литература»: Издательский дом «Федоров», 2012.
  2. Аргинская И.И. Сборник заданий по математике для самостоятельных, проверочных и контрольных работ в начальной школе. — Самара: Корпорация «Федоров», Издательство «Учебная литература», 2006.

Использованные изображения:

Спрашивает, любите ли вы математику.

Какие имена прилагательные характеризуют эту науку?

Какая, по-вашему, ещё эта наука?

Чей портрет на доске?

А знаете почему портрет М.В. Ломоносова у нас на уроке?

Он говорил: «Математику уже затем учить надо, что она ум в порядок приводит».

Так какая ещё эта наука?

Полагаясь на слова М.В. Ломоносова, будем изучать математику?

Предлагает озаглавить запись.

Предлагает решить уравнения, найти «лишнее» и доказать.

Спрашивает, как найти неизвестное слагаемое.

Предлагает ученику выполнить задание по карточке на доске самостоятельно.

А остальным учащимся предлагает

Игру « Да и Нет». (Презентация игры)

Предлагает озаглавить.

Спрашивает, что объединяет их.

Предлагает разделить уравнения на 2группы.

Предлагает объяснить в чём отличие уравнений,которые не решали, т.е. сложных.

Предлагает назвать тему урока и сформулировать задачу.

Спрашивает, что поможет научиться решать сложные уравнения.

Спрашивает, сможем из нового уравнения сделать простое, какое умеем решать и что для этого надо сделать.

А можем найти сумму? Как?

Объясняет, что в математике это называется упростить уравнение.

Спрашивает, а сумма может быть выражена частным чисел, разностью чисел, суммой чисел.

Организует работу в парах. Предлагает упорядочить алгоритм решения уравнения и определить, это алгоритм решения простого или сложного уравнения.

Предлагает обосновать ответ.

Предлагает проверить на доске.

Предлагает определить, какие это уравнения и объяснить решение уравнений, используя алгоритм.

Предлагает сравнить уравнения, распределить по степени сложности, решить более сложное по алгоритму на доске.

Предлагает решить задачу, составляя уравнение, используя алгоритм.

Предлагает построить шкалу знаний, оценить свои знания и умения, карандашом отметить их уровень:

1.Я знаю, что такое уравнение.

2.Я знаю, как решить простое уравнение на нахождение неизвестного слагаемого.

3.Я смогу упростить.

4.Я смогу решить усложнённое уравнение на нахождение неизвестного слагаемого.

Ставит учебную задачу: выбрать из трёх уравнений на карточке то уравнение, с которым ты считаешь справишься и решить его самостоятельно.

Предлагает проверку на доске.

Предлагает на шкале знаний зелёной ручкой показать, на каком вы уровне.

Спрашивает о возникших трудностях в решении.

Предлагает взять квадратик, который по цвету соответствует квадратику вашего уравнения на карточке, если уравнение решено верно. Если неправильно решили -возьмите квадратик коричневого цвета и давайте на доске построим диаграмму.

Предлагает оценить работу на уроке. Как вы думаете, достигли мы цели нашего урока? Научились решать усложнённые уравнения?

Спрашивает, что помогло справиться с решением уравнений.

Организует обсуждение выполнения домашней работы на с.62 «Выбери задание сам».

Решение уравнений, содержащих дроби, используя свойство умножения равенства

Результаты обучения

  • Используйте свойства умножения и деления для решения уравнений с дробями и делением

Решайте уравнения с дробями, используя свойства умножения и деления равенства

Мы будем решать уравнения, требующие умножения или деления для выделения переменной. Во-первых, давайте снова рассмотрим свойство разделения равенства.

Разделение собственности равенства

Для любых номеров [латекс] a, b [/ латекс] и [латекс] c [/ латекс],

[латекс] \ text {if} a = b, \ text {then} \ frac {a} {c} = \ frac {b} {c} [/ latex].
Если вы разделите обе части уравнения на одинаковую величину, вы все равно получите равенство.

Давайте применим эту идею на практике. Мы ищем число, которое вы умножаете на [latex] 10 [/ latex], чтобы получить [latex] 44 [/ latex], и мы можем использовать деление, чтобы узнать.

Пример

Решить: [латекс] 10q = 44 [/ латекс]

Показать решение

Решение:

[латекс] 10q = 44 [/ латекс]
Разделите обе стороны на [латекс] 10 [/ латекс], чтобы отменить умножение. [латекс] \ frac {10q} {10} = \ frac {44} {10} [/ латекс]
Упростить. [латекс] q = \ frac {22} {5} [/ латекс]
Чек:
Замените [latex] q = \ frac {22} {5} [/ latex] в исходное уравнение. [латекс] 10 \ left (\ frac {22} {5} \ right) \ stackrel {?} {=} 44 [/ latex]
Упростить. [латекс] \ stackrel {2} {\ overline {) 10}} \ left (\ frac {22} {\ overline {) 5}} \ right) \ stackrel {?} {=} 44 [/ latex]
Умножить. [латекс] 44 = 44 [/ латекс]

Решением уравнения была фракция [латекс] \ frac {22} {5} [/ latex]. Оставляем это как неправильную дробь.

Теперь рассмотрим уравнение [латекс] \ frac {x} {4} = 3 [/ latex]. Мы хотим знать, какое число, разделенное на [латекс] 4 [/ латекс], дает [латекс] 3 [/ латекс]. Итак, чтобы «отменить» деление, нам нужно будет умножить на [латекс] 4 [/ латекс]. Свойство умножения равенства позволит нам это сделать. Это свойство говорит о том, что если мы начнем с двух равных величин и умножим их на одно и то же число, результаты будут одинаковыми.

Свойство равенства умножения

Для любых номеров [латекс] a, b [/ латекс] и [латекс] c [/ латекс],

[латекс] \ text {if} a = b, \ text {then} ac = bc [/ latex].
Если вы умножите обе части уравнения на одинаковую величину, вы все равно получите равенство.

Давайте воспользуемся свойством умножения равенства, чтобы решить уравнение [latex] \ frac {x} {7} = — 9 [/ latex].

Пример

Решение: [latex] \ frac {x} {7} = — 9 [/ latex].

Решение:

[латекс] \ frac {x} {7} = — 9 [/ латекс]
Используйте свойство умножения равенства, чтобы умножить обе стороны на [латекс] 7 [/ латекс].Это изолирует переменную. [латекс] \ color {red} {7} \ cdot \ frac {x} {7} = \ color {red} {7} (- 9) [/ latex]
Умножить. [латекс] \ frac {7x} {7} = — 63 [/ латекс]
Упростить. [латекс] x = -63 [/ латекс]
Проверить.

Замените [латекс] \ color {red} {- 63} [/ latex] вместо x в исходном уравнении.

[латекс] \ frac {\ color {red} {- 63}} {7} \ stackrel {?} {=} — 9 [/ latex]
Уравнение верно. [латекс] -9 = -9 [/ латекс]

Пример

Решить: [латекс] \ frac {p} {- 8} = — 40 [/ латекс].

Показать решение

Решение:
Здесь [latex] p [/ latex] делится на [latex] -8 [/ latex]. Мы должны умножить на [латекс] -8 [/ латекс], чтобы выделить [латекс] п [/ латекс].

[латекс] \ frac {p} {- 8} = — 40 [/ латекс]
Умножить обе стороны на [латекс] -8 [/ латекс] [латекс] \ color {red} {- 8} (\ frac {p} {- 8}) = \ color {red} {- 8} (- 40) [/ latex]
Умножить. [латекс] \ frac {-8p} {- 8} = 320 [/ латекс]
Упростить. [латекс] p = 320 [/ латекс]
Чек:
Заменитель [латекс] р = 320 [/ латекс]. [латекс] \ frac {\ color {красный} {320}} {- 8} \ stackrel {?} {=} — 40 [/ латекс]
Уравнение верно. [латекс] -40 = -40 [/ латекс]

В следующем видео мы показываем еще два примера того, когда использовать свойства умножения и деления для решения одношагового уравнения.

Решите уравнения с коэффициентом [латекс] -1 [/ латекс]

Посмотрите на уравнение [латекс] -y = 15 [/ латекс]. Кажется, что [латекс] y [/ latex] уже изолирован? Но перед [latex] y [/ latex] стоит отрицательный знак, поэтому он не изолирован.
Существует три различных способа изолировать переменную в уравнении этого типа. В следующем примере мы покажем все три способа.

Пример

Решить: [латекс] -y = 15 [/ латекс].

Показать решение

Решение:
Один из способов решить это уравнение — переписать [latex] -y [/ latex] как [latex] -1y [/ latex], а затем использовать свойство равенства разделения для выделения [latex] y [/ latex] ].

[латекс] -y = 15 [/ латекс]
Перепишите [latex] -y [/ latex] как [latex] -1y [/ latex]. [латекс] -1y = 15 [/ латекс]
Разделите обе стороны на [латекс] -1 [/ латекс]. [латекс] \ frac {-1y} {\ color {red} {- 1}} = \ frac {15} {\ color {red} {- 1}} [/ latex]
Упростите каждую сторону. [латекс] y = -15 [/ латекс]

Другой способ решить это уравнение — умножить обе части уравнения на [латекс] -1 [/ латекс].

[латекс] y = 15 [/ латекс]
Умножьте обе стороны на [латекс] -1 [/ латекс]. [латекс] \ color {red} {- 1} (- y) = \ color {red} {- 1} (15) [/ latex]
Упростите каждую сторону. [латекс] y = -15 [/ латекс]

Третий способ решить уравнение — прочитать [latex] -y [/ latex] как «противоположность [latex] y \ text {. »} [/ Latex] Какое число имеет [latex] 15 [/ latex] как его противоположность? Противоположностью [латекса] 15 [/ латекса] является [латекс] -15 [/ латекс].Итак [латекс] y = -15 [/ латекс].
Для всех трех методов мы изолировали [латекс] y [/ латекс] изолировали и решили уравнение.
Чек:

[латекс] y = 15 [/ латекс]
Заменитель [латекс] y = -15 [/ латекс]. [латекс] — (\ color {red} {- 15}) \ stackrel {?} {=} (15) [/ латекс]
Упростить. Уравнение верно. [латекс] 15 = 15 [/ латекс]

В следующем видео мы покажем больше примеров того, как решить уравнение с отрицательной переменной.

Решите уравнения с дробным коэффициентом

Когда у нас есть уравнение с дробным коэффициентом, мы можем использовать свойство равенства умножения, чтобы сделать коэффициент равным [латекс] 1 [/ латекс].
Например, в уравнении:

[латекс] \ frac {3} {4} x = 24 [/ latex]
Коэффициент [латекса] x [/ latex] равен [latex] \ frac {3} {4} [/ latex]. Чтобы найти [latex] x [/ latex], нам нужно, чтобы его коэффициент был [latex] 1 [/ latex]. Поскольку произведение числа и его обратное значение равно [latex] 1 [/ latex], наша стратегия здесь будет заключаться в том, чтобы выделить [latex] x [/ latex] путем умножения на обратную величину [latex] \ frac {3} {4 }[/латекс].Мы сделаем это в следующем примере.

Пример

Решение: [латекс] \ frac {3} {4} x = 24 [/ latex].

Показать решение

Решение:

[латекс] \ frac {3} {4} x = 24 [/ латекс]
Умножьте обе части на обратное значение коэффициента. [латекс] \ color {красный} {\ frac {4} {3}} \ cdot \ frac {3} {4} x = \ color {красный} {\ frac {4} {3}} \ cdot 24 [ / латекс]
Упростить. [латекс] 1x = \ frac {4} {3} \ cdot \ frac {24} {1} [/ latex]
Умножить. [латекс] x = 32 [/ латекс]
Чек: [латекс] \ frac {3} {4} x = 24 [/ латекс]
Заменитель [латекс] x = 32 [/ латекс]. [латекс] \ frac {3} {4} \ cdot 32 \ stackrel {?} {=} 24 [/ латекс]
Записываем [латекс] 32 [/ латекс] дробью. [латекс] \ frac {3} {4} \ cdot \ frac {32} {1} \ stackrel {?} {=} 24 [/ латекс]
Умножить. Уравнение верно. [латекс] 24 = 24 [/ латекс]

Обратите внимание, что в уравнении [latex] \ frac {3} {4} x = 24 [/ latex] мы могли бы разделить обе стороны на [latex] \ frac {3} {4} [/ latex] получить сам [латекс] х [/ латекс].Деление аналогично умножению на обратное, поэтому результат будет тот же. Но большинство людей согласны с тем, что умножить на обратное проще.

Пример

Решить: [латекс] — \ frac {3} {8} w = 72 [/ латекс].

Показать решение

Решение:
Коэффициент — отрицательная дробь. Помните, что число и обратная величина имеют один и тот же знак, поэтому обратная величина коэффициента также должна быть отрицательной.

[латекс] \ frac {3} {8} w = 72 [/ латекс]
Умножьте обе стороны на обратное значение [латекс] — \ frac {3} {8} [/ latex]. [латекс] \ color {red} {- \ frac {8} {3}} (- \ frac {3} {8} w) = (\ color {red} {- \ frac {8} {3}} ) 72 [/ латекс]
Упростить; обратные умножаются на единицу. [латекс] 1w = — \ frac {8} {3} \ cdot \ frac {72} {1} [/ латекс]
Умножить. [латекс] w = -192 [/ латекс]
Чек: [латекс] — \ frac {3} {8} w = 72 [/ латекс]
Пусть [латекс] w = -192 [/ латекс]. [латекс] — \ frac {3} {8} (- 192) \ stackrel {?} {=} 72 [/ латекс]
Умножить.Это проверяет. [латекс] 72 = 72 [/ латекс]

В следующем видео-примере вы увидите еще один пример того, как использовать обратную величину дробного коэффициента для решения уравнения.

Устранение неравенств

Иногда нам нужно решить такие неравенства:

Символ

Слова

Пример

>

больше

х + 3 > 2

<

менее

7x < 28

больше или равно

5 x — 1

меньше или равно

2 года + 1 7

Решение

Наша цель — иметь x (или другую переменную) самостоятельно слева от знака неравенства:

Примерно так: х <5
или: г ≥ 11

Мы называем это «решенным».

Пример: x + 2> 12

Вычтем 2 с обеих сторон:

х + 2 — 2> 12 — 2

Упростить:

x> 10

Решено!

Как решить

Решение неравенств очень похоже на решение уравнений … мы делаем почти то же самое …

… но мы также должны обратить внимание на направление неравенства .

Направление: куда «указывает» стрелка

Некоторые вещи могут изменить направление !

<становится>

> становится <

≤ становится ≥

≥ становится ≤

Безопасные дела

Эти вещи не влияют на направление неравенства:

  • Сложить (или вычесть) число с обеих сторон
  • Умножение (или деление) обеих сторон на положительное число
  • Упростить сторону

Пример: 3x

<7 + 3

Мы можем упростить 7 + 3, не влияя на неравенство:

3x <10

Но эти вещи действительно изменяют направление неравенства (например, «<" становится ">«):

Пример: 2y + 7

<12

Когда мы меняем местами левую и правую части, мы также должны изменить направление неравенства :

12 > 2 года + 7

Вот подробности:

Сложение или вычитание значения

Мы часто можем решить неравенства, добавляя (или вычитая) число с обеих сторон (точно так же, как во Введении в алгебру), например:

Пример: x + 3

<7

Если вычесть 3 с обеих сторон, мы получим:

x + 3 — 3 <7 — 3

х <4

И вот наше решение: x <4

Другими словами, x может быть любым значением меньше 4.

Что мы сделали?

Мы пошли от этого:

К этому:

х + 3 <7

х <4

И это хорошо работает для , складывая и вычитая , потому что если мы прибавим (или вычтем) одинаковую сумму с обеих сторон, это не повлияет на неравенство

Пример: У Алекса больше монет, чем у Билли.Если и Алекс, и Билли получат по три монеты больше, у Алекс все равно будет больше монет, чем у Билли.

Что, если я решу, но «x» справа?

Неважно, просто поменяйте местами стороны, но переверните знак , чтобы он все еще «указывал» на правильное значение!

Пример: 12

Если вычесть 5 с обеих сторон, получим:

12 -5 -5

7 <х

Вот и решение!

Но ставить «x» слева — это нормально…

… так давайте обратим внимание (и знак неравенства!):

x> 7

Вы видите, как знак неравенства все еще «указывает» на меньшее значение (7)?

И вот наше решение: x> 7

Примечание: «x» может быть справа, но людям обычно нравится видеть его слева.

Умножение или деление на значение

Еще мы делаем умножение или деление обеих сторон на значение (как в алгебре — умножение).

Но нам нужно быть немного осторожнее (как вы увидите).

Положительные значения

Все нормально, если мы хотим умножить или разделить на положительное число :

Пример: 3y

<15

Если мы разделим обе стороны на 3, получим:

3 года /3 <15 /3

г <5

И вот наше решение: y <5

Отрицательные значения

Когда мы умножаем или делим на отрицательное число
, мы должны обратить неравенство.

Почему?

Ну, посмотрите на числовую строку!

Например, от 3 до 7 — это , увеличение ,
, но от -3 до -7 — это , уменьшение.

−7 <−3 7> 3

Видите, как меняет знак неравенства (с <на>)?

Давайте попробуем пример:

Пример: −2y

<−8

Разделим обе части на −2… и отменяют неравенство !

−2y <−8

−2y / −2 > −8 / −2

г> 4

И это правильное решение: y> 4

(Обратите внимание, что я перевернул неравенство в той же строке , которую я разделил на отрицательное число.)

Итак, запомните:

При умножении или делении на отрицательное число отменяет неравенство

Умножение или деление на переменные

Вот еще один (хитрый!) Пример:

Пример: bx

<3b

Кажется легко просто разделить обе стороны на b , что дает нам:

x <3

… но подождите … если b является отрицательным , нам нужно изменить неравенство следующим образом:

x> 3

Но мы не знаем, положительное или отрицательное значение b, поэтому мы не можем ответить на этот вопрос !

Чтобы помочь вам понять, представьте, что замените b на 1 или −1 в примере bx <3b :

  • , если b равно 1 , то ответ будет x <3
  • , но если b равно −1 , то мы решаем −x <−3 , и ответ будет x> 3

Ответом может быть x <3 или x> 3 , и мы не можем выбрать, потому что не знаем b .

Так:

Не пытайтесь делить на переменную, чтобы решить неравенство (если вы не знаете, что переменная всегда положительна или всегда отрицательна).

Пример побольше

Пример:

x − 3 2 <−5

Во-первых, давайте очистим «/ 2», умножив обе стороны на 2.

Поскольку мы умножаем на положительное число, неравенства не изменятся.

x − 3 2 × 2 <−5 × 2

х-3 <-10

Теперь прибавьте 3 к обеим сторонам:

x − 3 + 3 <−10 + 3

х <−7

И вот наше решение: x <−7

Два неравенства сразу!

Как решить что-то сразу с двумя неравенствами?

Пример:

−2 < 6−2x 3 <4

Во-первых, давайте очистим «/ 3», умножив каждую часть на 3.

Поскольку мы умножаем на положительное число, неравенства не меняются:

−6 <6−2x <12

Теперь вычтите 6 из каждой части:

−12 <−2x <6

Теперь разделите каждую часть на 2 (положительное число, чтобы неравенства снова не изменились):

−6 <−x <3

Теперь умножьте каждую часть на -1. Поскольку мы умножаем на отрицательное число , неравенства меняют направление .

6> х> −3

И это решение!

Но для наглядности лучше иметь меньшее число слева, большее — справа. Так что давайте поменяем их местами (и убедимся, что неравенства указаны правильно):

−3 <х <6

Сводка

  • Многие простые неравенства могут быть решены путем сложения, вычитания, умножения или деления обеих сторон, пока не останется переменная сама по себе.
  • Но вот эти вещи изменят направление неравенства:
    • Умножение или деление обеих сторон на отрицательное число
    • Замена левой и правой сторон
  • Не умножайте и не делите на переменную (если вы не знаете, что она всегда положительна или всегда отрицательна)

Комплексные числа: умножение

Комплексные числа: умножение

Умножение производится алгебраически.

Сложное умножение — это более сложная операция для понимания с алгебраической или геометрической точки зрения. Давайте сначала сделаем это алгебраически, а для умножения возьмем определенные комплексные числа, например 3 + 2 i и 1 + 4 i. В каждом есть два члена, поэтому, когда мы их умножим, мы получим четыре члена:

(3 + 2 i ) (1 + 4 i ) =
3 + 12 i + 2 i + 8 i 2 .

Теперь 12 i + 2 i упрощается до 14 i, конечно же, .А как насчет 8 i 2 ? Помните, что мы ввели i как сокращение для √ – 1, квадратного корня из –1. Другими словами, i — это что-то, квадрат которого равен –1. Таким образом, 8 i 2 равно –8. Следовательно, произведение (3 + 2 i ) (1 + 4 i ) равно –5 + 14 i.

Если вы обобщите этот пример, вы получите общее правило умножения

Помните, что ( xu yv ), действительная часть продукта, является произведением реальных частей минус произведение мнимых частей, но ( xv + yu ) мнимая часть продукт, представляет собой сумму двух произведений одной действительной части и другой мнимой части.

Давайте посмотрим на некоторые частные случаи умножения.

Умножение комплексного числа на действительное

В приведенной выше формуле умножения, если v равно нулю, вы получите формулу для умножения комплексного числа x + yi и действительного числа u вместе:

( x + yi ) u =
xu + yu i .

Другими словами, вы просто умножаете обе части комплексного числа на действительное число.Например, 2 умножить на 3 + i будет просто 6 + 2 i. Геометрически, когда вы удваиваете комплексное число, просто удваиваете расстояние от начала координат, 0. Точно так же, когда вы умножаете комплексное число z на 1/2, результат будет посередине между 0 и z. Умножение на 2 можно рассматривать как преобразование, которое растягивает комплексную плоскость C с коэффициентом 2 от 0; и умножение на 1/2 как преобразование, которое сжимает C в сторону 0.

Умножение и абсолютное значение.

Несмотря на то, что мы сделали только один случай для умножения, достаточно предположить, что абсолютное значение zw (то есть расстояние от 0 до zw ) может быть абсолютным значением z , умноженным на абсолютное значение . ш. Это было тогда, когда w было действительным числом u чуть выше. На самом деле это так в целом:

Проверка этого тождества — это упражнение по алгебре.Чтобы доказать это, мы докажем, что это верно для квадратов, поэтому нам не придется иметь дело с квадратными корнями. Мы покажем | zw | 2 = | z | 2 | w | 2 . Пусть z будет x + yi, и пусть w будет u + vi. Тогда согласно формуле умножения zw равно ( xu yv ) + ( xv + yu ) i. Вспомните из раздела об абсолютных величинах, что

| z | 2 = x 2 + y 2

Аналогично имеем

| w | 2 = u 2 + v 2

и, поскольку zw = ( xu yv ) + ( xv + yu ) i,

| wz | 2 = ( xu yv ) 2 + ( xv + yu ) 2

Итак, чтобы показать | zw | 2 = | z | 2 | w | 2 , все, что вам нужно сделать, это показать, что

( xu yv ) 2 + ( xv + yu ) 2 = ( x 2 + y 2 ) ( u 21 + v 2 )

и это простое упражнение по алгебре.

Полномочия

i.

В следующем частном случае умножения рассмотрим различные степени мнимой единицы i. Мы начали с предположения, что i 2 = –1. Что насчет i 3 ? Это всего лишь i 2 умноженное на i , то есть –1 умноженное на i. Следовательно, i 3 = — i. Это интересно: куб i — это собственное отрицание.Затем рассмотрим i 4 . Это квадрат i 2 , то есть квадрат –1. Итак, i 4 = 1. Другими словами, i — это корень четвертой степени из 1. Вы можете показать, что — i — это еще один корень четвертой степени из 1. И поскольку и –1, и 1 являются квадратными корнями из 1, теперь мы знаем все четыре корня четвертой степени из 1, а именно,
1, i, –1 и — i. Это наблюдение связано с фундаментальной теоремой алгебры, поскольку уравнение z 4 = 1 является уравнением четвертой степени, поэтому должно иметь ровно четыре корня.

Более высокие степени i легко найти теперь, когда мы знаем i 4 = 1. Например, i 5 равно i , умноженное на i 4 , и это всего лишь i. . Вы можете уменьшить мощность i на 4 и не изменять результат. Другой пример: i 11 = i 7 = i 3 = — i.

Как насчет отрицательной степени и ? Что является обратным для i,
то есть i –1 ? По той же причине, что вы можете вычесть 4 из степени i и не изменить результат, вы также можете прибавить 4 к степени i. Это означает i –1 = i 3 = — i. Таким образом, i обратное — i. Представьте себе — число, обратное значение которого — собственное отрицание! Конечно, легко проверить, что i раз — i равно 1, поэтому, конечно,
i и — i являются обратными величинами.


Корни единства.

Различные корни из 1 называются корнями из единицы. В общем, по Фундаментальной теореме алгебры число n корней -й степени из единицы равно n, , так как существует n корней у уравнения n -й степени
z u — 1 = 0.Квадратные корни из единицы равны 1 и –1. Корни четвертой степени равны ± 1, ± i, , как отмечалось ранее в разделе, посвященном абсолютным значениям. Кроме того, в этом разделе упоминалось, что ± √2 / 2 ± i √2 / 2 были квадратными корнями из i и — i, и теперь с формулой умножения, которую легко проверить. Следовательно, восемь корней из восьми из единицы равны ± 1, ± i, и ± √2 / 2 ± i √2 / 2. Обратите внимание на то, как эти восемь корней единицы равномерно распределены по единичной окружности.

Мы можем использовать геометрию, чтобы найти другие корни из единицы, в частности кубические корни и корни шестой степени из единицы. Но давайте их немного подождем.


Умножение комплексного числа на

i.

В нашей цели по поиску геометрической интерпретации комплексного умножения, давайте теперь рассмотрим умножение произвольного комплексного числа z = x + yi на i.

z i = ( x + yi ) i =
y + xi .

Давайте интерпретируем это утверждение геометрически. Точка z в C расположена на x единиц справа от мнимой оси и на y единиц выше действительной оси. Точка z i расположена на y единиц слева и x единиц выше. Произошло то, что при умножении на i повернулся к точке z на 90 ° против часовой стрелки вокруг начала координат до точки z i. Короче говоря, умножение на i дает поворот на 90 ° против часовой стрелки на 0.

Вы можете проанализировать, что происходит при умножении на — i таким же образом. Вы обнаружите, что умножение на — i дает поворот на 90 ° по часовой стрелке примерно на 0. Когда мы не указываем против часовой стрелки или по часовой стрелке при обращении к поворотам или углам, мы будем следовать стандартному соглашению, которое подразумевается против часовой стрелки. Тогда мы можем сказать, что умножение на — i дает поворот на –90 ° вокруг 0 ​​или, если хотите, поворот на 270 ° вокруг 0.

Геометрическая интерпретация умножения.

Чтобы полностью оправдать то, что мы собираемся увидеть, необходима тригонометрия, и это делается в необязательном разделе. А пока посмотрим на результаты без обоснования. Мы видели два особых случая умножения: один на вещественные числа, что приводит к масштабированию, другой на i , что приводит к вращению. Общий случай — это комбинация масштабирования и вращения.

Пусть z и w — точки на комплексной плоскости C .Проведите линии от 0 до z и от 0 до w . Длины этих строк — абсолютные значения | z | и | w | соответственно. Мы уже знаем, что длина строки от 0 до zw будет абсолютным значением | zw | что равно | z | | w |. (На диаграмме | z | составляет около 1,6, а | w | составляет около 2,1, поэтому | zw | должно быть около 3,4. Обратите внимание, что единичный круг закрашен.) Чего мы не знаем, так это направления линии от 0 до zw.

Ответ: «углы складываются». Мы определим направление линии от 0 до z по определенному углу, называемому аргументом из z , иногда обозначаемым arg ( z ). Это угол, вершина которого равна 0, первая сторона — положительная действительная ось, а вторая сторона — прямая от 0 до z. Другая точка w имеет угол arg ( w ).Тогда произведение zw будет иметь угол, который является суммой углов arg ( z ) + arg ( w ). (На диаграмме arg ( z ) составляет около 20 °, а arg ( w ) составляет около 45 °, поэтому arg ( zw ) должно быть около 65 °.)

Таким образом, у нас есть два уравнения, которые определяют, где находится zw в C :

Свойства равенств (Алгебра 1, Как решать линейные уравнения) — Mathplanet

Два уравнения, которые имеют одно и то же решение, называются эквивалентными уравнениями e.грамм. 5 +3 = 2 + 6. И это, как мы узнали в предыдущем разделе, обозначается знаком равенства =. Обратная операция — это две операции, которые отменяют друг друга, например сложение и вычитание или умножение и деление. Вы можете выполнить ту же обратную операцию с каждой стороной эквивалентного уравнения, не меняя равенства.

$$ 5 + 3 \, {\ color {green} {- \, 2}} = 6 + 2 \, {\ color {green} {- \, 2}} $$

Это дает нам несколько свойств, которые верны для всех уравнений.

Свойство сложения равенства говорит нам, что добавление одного и того же числа к каждой стороне уравнения дает нам эквивалентное уравнение

$$, если \: ab = c, то \: ab \, {\ color {green} {+ \, b}} = c \, {\ color {green} {+ \, b}} или \: а = с + b $$

То же самое и со свойством вычитания равенства .

$$ если \: a + b = c, то \: a + b \, {\ color {green} {- \, b}} = c \, {\ color {green} {- \, b}} , или \: a = cb $$

Так же, как и свойство умножения равенства . Если вы умножите каждую сторону уравнения на одно и то же ненулевое число, вы получите эквивалентное уравнение.

$$, если \: \ frac {a} {b} = c и \: b \ neq 0, то \: \ frac {a} {b} \, \ cdot {\ color {green} b} = c \ cdot \, {\ color {green} b} или \: a = cb $$

И, естественно, это касается и свойства разделения равенства .Вы можете разделить каждую часть уравнения на одно и то же ненулевое число, чтобы получить эквивалентное уравнение

$$ если \: a \ cdot b = c и \: b \ neq 0, то \: \ frac {a \ cdot b} {{\ color {green} b}} \, = \ frac {c} {{\ color {green} b}} или \: a = \ frac {c} {b} $$

Это дает нам возможность изменить уравнение по своему усмотрению. Приемлемо все, что угодно, если вы делаете одно и то же с обеих сторон.

Есть несколько других свойств уравнений, о которых также может быть полезно знать


Пример

Джордж срубил дуб высотой 60 футов.Теперь он хочет разрезать его на более мелкие кусочки. Сначала он разрезает его на две части, каждая по 30 футов. А затем он продолжает делать десять деталей длиной 6 футов, прежде чем погрузить их в свой грузовик.

Глядя на разные куски дерева, мы можем убедиться в следующем.

$$ 60 = 30 + 30 = 6 + 6 + 6 + 6 + 6 + 6 + 6 + 6 + 6 + 6 $$

Это называется рефлексивным свойством равенства и говорит нам, что любая величина равна самой себе

$$ a = a $$

Мы также можем использовать этот пример с кусками дерева, чтобы объяснить свойство симметрии равенства .Это свойство гласит, что если количество a равно количеству b, то b равно a.

$$, если \: a = b, \: then \: b = a $$

Или, если использовать наш пример

$$ если \: 60 = 30 + 30, \: то \: 30 + 30 = 60 $$

Еще одно свойство, которое можно объяснить этим, — это переходное свойство равенства . Он говорит нам, что если количество a равно количеству b, а b равно количеству c, то a и c также равны.

$$ if \: a = b \: and \: b = c, \: then \: a = c $$

Или в числах, взятых из примера дуба

$$ if \: 60 = 30 + 30 $$

$$ и \: 30 + 30 = 6 + 6 + 6 + 6 + 6 + 6 + 6 + 6 + 6 + 6 $$

$$, тогда \: 60 = 6 + 6 + 6 + 6 + 6 + 6 + 6 + 6 + 6 + 6 $$

Поскольку мы знаем, что 30 + 30 = 20 + 40 и 30 + 30 = 60, мы можем заменить 30 + 30 на 20 + 40 и получить 60 = 20 + 40.Это называется свойством замещения равенства .

Если a = b, то a можно заменить на b в любом выражении.


Видеоурок

Решите эти уравнения, используя обратные операции

$$ x + 8 = 10 $$

$$ x — 4 = 22 $$

$$ x \ div 3 = 6 $$

$$ 7x = 28 $$

Решение рациональных уравнений

Решение рациональных уравнений

Рациональное уравнение Уравнение, содержащее по крайней мере одно рациональное выражение.- уравнение, содержащее хотя бы одно рациональное выражение. Рациональные выражения обычно содержат переменную в знаменателе. По этой причине мы позаботимся о том, чтобы знаменатель не был равен нулю, отметив ограничения и проверив наши решения.

Решите рациональные уравнения, удаляя дроби, умножая обе части уравнения на наименьший общий знаменатель (LCD).

Пример 1: Решите: 5x − 13 = 1x.

Решение: Сначала отметим, что x ≠ 0, а затем умножим обе стороны на ЖК-дисплей, 3 x :

Проверьте свой ответ, заменив 12 на x , чтобы убедиться, что вы получили истинное утверждение.

Ответ: Решение — 12.

После умножения обеих частей предыдущего примера на ЖК-дисплей, нам осталось решить линейное уравнение.Это не всегда так; иногда нам остается квадратное уравнение.

Пример 2: Решите: 2−1x (x + 1) = 3x + 1.

Решение: В этом примере есть два ограничения: x ≠ 0 и x ≠ −1. Начните с умножения обеих сторон на ЖК-дисплей, x (x + 1).

После распределения и деления общих множителей остается квадратное уравнение. Чтобы решить эту проблему, перепишите его в стандартной форме, коэффициент, а затем установите каждый коэффициент равным 0.

Убедитесь, что эти значения решают исходное уравнение.

Ответ: Решения -1/2 и 1.

До этого момента все возможные решения решали исходное уравнение. Однако так бывает не всегда. Умножение обеих частей уравнения на переменные множители может привести к посторонним решениям. Решение, которое не решает исходное уравнение, то есть решения, которые не решают исходное уравнение.Полный список шагов для решения рационального уравнения представлен в следующем примере.

Пример 3: Решите: xx + 2 + 2×2 + 5x + 6 = 5x + 3.

Раствор:

Шаг 1: Разложите все знаменатели на множители и определите ЖК-дисплей.

ЖК-дисплей равен (x + 2) (x + 3).

Шаг 2: Определите ограничения. В данном случае это x ≠ −2 и x ≠ −3.

Шаг 3: Умножьте обе части уравнения на ЖК-дисплей. Распространяйте осторожно, а затем упрощайте.

Шаг 4: Решите полученное уравнение. Результатом является квадратное уравнение. Перепишите его в стандартной форме с коэффициентом, а затем установите каждый коэффициент равным 0.

Шаг 5: Проверьте наличие посторонних решений. Всегда подставляйте в исходное уравнение или его факторизованный эквивалент.В этом случае выберите факторизованный эквивалент для проверки:

Здесь −2 — постороннее решение, не входящее в набор решений. Важно отметить, что −2 — это ограничение.

Ответ: Решение 4.

Если этот процесс приводит к решению, которое является ограничением, не считайте его посторонним решением.

Попробуй! Решите: xx − 5 + 3x + 2 = 7xx2−3x − 10.

Ответ: −3

Иногда все потенциальные решения являются посторонними, и в этом случае мы говорим, что не существует решения исходного уравнения. В следующих двух примерах мы демонстрируем два способа, по которым рациональное уравнение не может иметь решений.

Пример 4: Решить: 3xx2−4−2x + 2 = 1x + 2.

Решение: Чтобы идентифицировать ЖК-дисплей, сначала разложите знаменатели на множители.

Умножьте обе стороны на наименьший общий знаменатель (LCD), (x + 2) (x − 2), аккуратно распределив.

Уравнение противоречит и поэтому не имеет решения.

Ответ: Нет решения, ∅

Пример 5: Решите: xx − 4−4x + 5 = 36×2 + x − 20.

Решение: Сначала разложите знаменатели на множители.

Обратите внимание, что ограничения x ≠ 4 и x −5. Чтобы очистить дроби, умножьте на ЖК-дисплей (x − 4) (x + 5).

Оба эти значения являются ограничениями исходного уравнения; следовательно, оба посторонние.

Ответ: Нет решения, ∅

Попробуй! Решите: 1x + 1 + xx − 3 = 4xx2−2x − 3.

Ответ: ∅

Важно отметить, что этот метод очистки алгебраических дробей работает только для уравнений. Не пытайтесь очищать алгебраические дроби при упрощении выражений. Напоминаем, что у нас

Необходимо упростить выражения и решить уравнения.Если мы умножим выражение на ЖК-дисплей, x (2x + 1), мы получим другое выражение, которое не эквивалентно.

Буквенные уравнения

Буквальные уравнения или формулы часто являются рациональными уравнениями. Следовательно, методы, описанные в этом разделе, могут использоваться для решения конкретных переменных. Предположим, что все выражения переменных в знаменателе отличны от нуля.

Пример 6: Решите относительно x : z = x − 5y.

Решение: Цель — выделить x . Предполагая, что y не равно нулю, умножьте обе стороны на y , а затем прибавьте 5 к обеим сторонам.

Ответ: x = yz + 5

Пример 7: Решите относительно c : 1c = 1a + 1b.

Решение: В этом примере цель состоит в том, чтобы изолировать c . Мы начинаем с умножения обеих сторон на ЖК-дисплей, a⋅b⋅c, осторожно распределяя.

В правой части уравнения вычтем за скобки c .

Затем разделите обе части уравнения на величину (b + a).

Ответ: c = abb + a

Попробуй! Решите относительно y : x = y + 1y − 1.

Ответ: y = x + 1x − 1

Ключевые выводы

  • Начните решать рациональные уравнения с умножения обеих частей на ЖК-дисплей.Полученное эквивалентное уравнение можно решить, используя методы, изученные до этого момента.
  • Умножение обеих частей рационального уравнения на выражение переменной вводит возможность посторонних решений. Следовательно, мы должны проверять решения на соответствие множеству ограничений. Если решение является ограничением, то оно не является частью домена и является посторонним.
  • При умножении обеих частей уравнения на выражение, аккуратно распределите и умножьте каждый член на это выражение.
  • Если все полученные решения являются посторонними, то исходное уравнение не имеет решений.

Тематические упражнения

Часть A: Рациональные уравнения

Решить.

1. 12 + 1x = 18

2. 13−1x = 29

3. 13x − 23 = 1x

4. 25x − 1x = 310

5. 12x + 1 = 5

6.33x − 1 + 4 = 5

7. 2x − 3x + 5 = 2x + 5

8. 5x2x − 1 = x − 12x − 1

9. 5x − 7 = 6x − 9

10. 5x + 5 = 3x + 1

11. x6−6x = 0

12. 5x + x5 = −2

13. хх + 12 = 2х

14. 2xx + 5 = 16 − x

15. 1x + x2x + 1 = 0

16. 9x3x − 1−4x = 0

17. 1−2x = 48×2

18. 2−9x = 5×2

19.1 + 12x = 12x − 2

20. 1−3x − 5x (3x − 4) = — 1x

21. x2 = 14x + 3

22. 3×2 = x + 13 − x

23. 6 = −3x + 3x − 1

24. 12x − 2 = 2 + 6 (4 − x) x − 2

25. 2 + 2xx − 3 = 3 (x − 1) x − 3

26. xx − 1 + 16x − 1 = x (x − 1) (6x − 1)

27. 12×2−81 = 1x + 9−2x − 9

28. 14×2−49 = 2x − 7−3x + 7

29. 6xx + 3 + 4x − 3 = 3xx2−9

30.3xx + 2−17x − 2 = −48×2−4

31. х − 1 + 3 = 0

32. 4 − y − 1 = 0

33. y − 2−4 = 0

34. 9x − 2−1 = 0

35,3 (x − 1) −1 + 5 = 0

36,5−2 (3x + 1) −1 = 0

37. 3 + 2x − 3 = 2x − 3

38. 1x = 1x + 1

39. хх + 1 = х + 1x

40. 3x − 13x = xx + 3

41. 4x − 7x − 5 = 3x − 2x − 5

42. xx2−9 = 1x − 3

43.3x + 4x − 8−28 − x = 1

44. 1x = 6x (x + 3)

45. 3x = 1x + 1 + 13x (x + 1)

46. xx − 1−34x − 1 = 9x (4x − 1) (x − 1)

47. 1x − 4 + xx − 2 = 2×2−6x + 8

48. xx − 5 + x − 1×2−11x + 30 = 5x − 6

49. xx + 1−65×2 + 4x − 1 = −55x − 1

50. −8×2−4x − 12 + 2 (x + 2) x2 + 4x − 60 = 1x + 2

51. хх + 2−20×2 − x − 6 = −4x − 3

52. х + 7x − 1 + x − 1x + 1 = 4×2−1

53.x − 1x − 3 + x − 3x − 1 = −x + 5x − 3

54. x − 2x − 5 − x − 5x − 2 = 8 − xx − 5

55. x + 7x − 2−81×2 + 5x − 14 = 9x + 7

56. xx − 6 + 1 = 5x + 3036 − x2

57. 2xx + 1−44x − 3 = −74×2 + x − 3

58. x − 5x − 10 + 5x − 5 = −5xx2−15x + 50

59. 5×2 + 5x + 4 + x + 1×2 + 3x − 4 = 5×2−1

60. 1×2−2x − 63 + x − 9×2 + 10x + 21 = 1×2−6x − 27

61. 4×2−4 + 2 (x − 2) x2−4x − 12 = x + 2×2−8x + 12

62. x + 2×2−5x + 4 + x + 2×2 + x − 2 = x − 1×2−2x − 8

63.6xx − 1−11x + 12×2 − x − 1 = 6x2x + 1

64. 8x2x − 3 + 4x2x2−7x + 6 = 1x − 2

Часть B: Буквальные уравнения

Решите для указанной переменной.

65. Решите относительно r : t = Dr.

66. Решить относительно b : h = 2Ab.

67. Решите для P : t = IPr.

68. Решить относительно π: r = C2π.

69. Решите относительно c : 1a = 1b + 1c.

70. Решите относительно y : m = y − y1x − x1.

71. Решите относительно w : P = 2 (l + w).

72. Решите относительно t : A = P (1 + rt).

73. Решите относительно м. : s = 1n + m.

74. Решить относительно S : h = S2πr − r.

75. Решите относительно x : y = xx + 2.

76. Решите относительно x : y = 2x + 15x.

77.Решите относительно R : 1R = 1R1 + 1R2.

78. Решите относительно S1: 1f = 1S1 + 1S2.

Часть C: Обсуждение

79. Объясните, почему умножение обеих частей уравнения на ЖК-дисплей иногда дает посторонние решения.

80. Объясните связь между методом перекрестного умножения и умножением обеих частей рационального уравнения на ЖК-дисплее.

81. Объясните, как мы можем отличить рациональное выражение от рационального уравнения.Как мы относимся к ним по-другому?

ответы

1: −8/3

3: -1

5: -2/5

7: 5/2

9: −3

11: −6, 6

13: −4, 6

15: -1

17: −6, 8

19: −4, 6

21: −7, 4

23:

25:

27: −39

29: 4/3, 3/2

31: -1/3

33: -1/2, 1/2

35: 2/5

37:

39: -1/2

41:

43: −7

45: 5

47: -1

49:

51: −4

53: 5/3

55:

57: 1/2

59: −6, 4

61: 10

63: 1/3

65: r = Dt

67: P = Itr

69: c = abb − a

71: ш = P − 2l2

73: m = 1 − sns

75: х = 2y1 − y

77: R = R1R2R1 + R2

Перемножение крестиком для решения уравнений с дробями

Дроби не так сложно, как вы думаете.Если вы хотите научиться решать уравнения с дробями путем перекрестного умножения, например, решая «x / 7 = 2/3» и «7/3 = 2 / x» относительно x, вы находитесь в нужном месте.

Решение уравнений: Золотое правило

Цель решения уравнения для x состоит в том, чтобы закончить с утверждением, в котором x равно чему-то, что не включает x.

Золотое правило для решения уравнений — применять одну и ту же операцию к обеим сторонам уравнения. Это может включать сложение, вычитание или умножение обеих сторон на одинаковую величину.

Например, если x / 37 = 5, вы можете умножить обе стороны на 37. Поскольку x, разделенное на 37, умноженное на 37, просто равно x, тогда x = 5 * 37, что = 185.

Решение уравнений с выборочными долями

Вот пример уравнения и пошаговый процесс его решения:

Пример уравнения: x / 7 = 2/3

Шаг первый: умножьте обе стороны на 7.

x / 7 * 7 = 2/3 * 7

Умножьте 2 и 7, чтобы получить 14.

x / 7 * 7 = 14/3

Отменить / 7 * 7, чтобы оставить x, и преобразовать в решение относительно x.

х = 14/3 = 4 2/3

Вы умножаете обе части на 7, чтобы исключить знаменатель в левой части уравнения. Семерки сокращаются слева. Затем вы решаете уравнение относительно x. В качестве последнего шага вы конвертируете неправильную дробь 14/3 в смешанную дробь 4 и 2/3.

Обратите внимание, что вы просто перемножили 7 в знаменателе. Если бы вы начали с 23 * x / 7 = 2/3, вы бы крестом умножили обе стороны на 7/23 вместо 7. Это оставило бы только x с левой стороны.

Смешанные фракции: еще один пример

Пример специального перекрестного умножения

: Изображение Майка ДеХаана

Во втором примере «7/3 = 2 / x» смешанная дробь уже преобразована из 2 1/3 в 7/3. Намного проще решать уравнения с дробями, содержащие неправильные дроби, чем уравнения со смешанными дробями.

Обратите внимание, что в этом примере в знаменателе стоит x. Вы должны явно указать предположение, что x не равен нулю.

Пример уравнения: 7/3 = 2 / x

Умножьте обе стороны на x, чтобы x был в числителе.

7/3 * x = 2 / x * x

Сократить / x * x.

7 * x / 3 = 2

Умножьте обе стороны на 3/7, чтобы сократить 7/3.

x * (7/3) * (3/7) = 2 * (3/7)

Сократите 7/3 на 3/7 и умножьте, чтобы найти x.

х = 2 * 3/7 = 6/7

Если перемножить все за один шаг, получится:

7/3 = 2 / x
(7/3) * (3 * x / 7) = (2 / x) * (3 * x / 7)
x = 6/7

Перекрестное умножение легче распознать, когда числители и знаменатели проходят через знак равенства при умножении.

Конкретный пример перекрестного умножения: изображение Майка ДеХаана

Решение общих уравнений с дробями

Затем попробуйте решить дроби в общем случае, когда «a», «b», «c» и «d» ненулевые целые числа. Опять же, решите относительно X, начиная с числителя или знаменателя.

Х / б = в / д

Умножаем обе стороны на b

X / b * b = c / d * b

Удалите / b * b в левой части и объедините правую часть уравнения.

X = (b * c) / d

В аналогичном случае перекрестное умножение было бы более очевидным:

a * X / b = c / d

Умножить обе стороны на b / a

a * X / b * b / a = c / d * b / a

Вычеркните «a / b * b / a» в левой части уравнения и свяжите числитель («c» и «b») и знаменатель («d» и «a») справа.

Х = с * б / д * а

Следующий пример начинается с X в знаменателе и решает самый сложный пример: c / d = a / (b * X).

c / d = a / (b * X)

Умножьте обе стороны на X * (d / c).

(c / d) * X * (d / c) = a / (b * X) * X * (d / c)

Отмените «c / d * d / c» слева и «/ X * X» справа.

Х = (а * г) / (б * в)

Пятиступенчатая сводка для решения уравнений с дробями

Простейшее перекрестное умножение дробей. Изображение Майка ДеХаана

Из приведенных выше примеров вы видели, как умножать дроби крестом: возьмите знаменатель одной дроби и умножьте обе части уравнения на это значение.

Одна часть уравнения упрощается, поскольку знаменатель сокращается, а другая часть увеличивается в том же соотношении, поэтому уравнение остается сбалансированным.

Пять основных шагов для решения уравнений с дробями:

  1. При необходимости замените любые смешанные фракции на неправильные. Например, измените 2 1/3 на 7/3.
  2. Определите простейший метод выделения неизвестного x только в одну сторону уравнения.
  3. Перекрестное умножение обеих сторон на одинаковые значения.Например, выберите знаменатель с одной стороны, который будет числителем нового множителя с обеих сторон.
  4. Если вам нужно упростить уравнение пошагово, сделайте это, повторяя шаг 3 по мере необходимости.
  5. При необходимости замените оставшиеся неправильные фракции на смешанные.

Не забывайте применять пошаговый подход, когда вы учитесь решать уравнение; со временем вы сможете быстро решать уравнения дробей.

Дополнительная справка по перекрестному умножению уравнений на дроби

Возможно, вам потребуется дополнительная помощь или попрактиковаться в решении уравнений с дробями или перекрестным умножением.Если вы изучаете математику в школе, первое предложение — попросить учителя помочь вам научиться решать задачи с дробями. Кроме того, вы можете найти репетитора по математике в Интернете или по объявлениям в местных газетах. Онлайн-репетиторство по математике или онлайн-упражнения с таблицами дробей могут быть рентабельными способами улучшить свои математические навыки.

Номер ссылки

Обучение математике онлайн. Перемножение крестиком . По состоянию на 7 февраля 2013 г.

Майк ДеХаан применяет свою степень бакалавра математики в области компьютерных наук, годы программирования на языке Cobol и контроля качества (включая тестирование расчета процентов по кредитным картам) для исследования и представления математической теории для непрофессионала.

Здесь, в Decoded Science, Майк занимается математикой. Он изучает основы математической теории, раскрывает парадоксы, применяет вычисления к популярным фильмам и сообщает о математических новостях.
Майк начал профессионально писать в 2010 году как единственный владелец DeHaan Services.

Найдите Майка в Google+

Уравнения с дробями — Полный курс алгебры

24

Очистка от фракций

2-й уровень

Чтобы решить уравнение с дробями, мы преобразуем его в уравнение без дробей, которое мы умеем решать.Методика называется очисткой от фракций.

Пример 1. Решите относительно x :

x
3
+ x — 2
5
= 6.

Решение . Очистить следующие дроби:

Умножьте обе части уравнения — каждый член — на НОК знаменателей.Тогда каждый знаменатель разделит на кратное. Тогда у нас будет уравнение без дробей.

НОК 3 и 5 равно 15. Следовательно, умножьте обе части уравнения на 15.

15 · x
3
+ 15 · x — 2
5
= 15 · 6

Слева распределите по 15 на каждый член.Теперь каждый знаменатель разделится на 15 — вот в чем суть — и мы получим следующее простое уравнение, «очищенное» от дробей:

5 x + 3 ( x -2) = 90.
Легко решается следующим образом:
5 x + 3 x — 6 = 90
8 x = 90 + 6
x = 96
8
= 12.

Мы говорим «умножить» обе части уравнения, но мы пользуемся тем фактом, что порядок, в котором мы умножаем или делим, не имеет значения. (Урок 1.) Поэтому мы сначала делим НОК на каждый знаменатель и таким образом очищаем от дробей.

Мы выбираем , кратное каждого знаменателя, потому что каждый знаменатель будет тогда его делителем.

Пример 2. Очистите дроби и решите относительно x :

x
2
5 x
6
= 1
9

Решение .НОК 2, 6 и 9 равно 18. (Урок 23 по арифметике). Умножьте обе части на 18 — и отмените.

9 x — 15 x = 2.

Нет необходимости писать 18. Ученик должен просто посмотреть на и увидеть, что 2 войдет в 18 девять (9) раз. Таким образом, этот член становится 9 x .

Затем посмотрите и увидите, что 6 переходит в 18 три раза (3).Таким образом, этот член становится 3 · −5 x = −15 x .

Наконец, посмотрите и увидите, что 9 превратится в 18 два (2) раза. Таким образом, этот член становится 2 · 1 = 2.

Вот очищенное уравнение и его решение:

9 x -15 x = 2
−6 x = 2
x = 2
−6
x = 1
3

Пример 3.Решить относительно x :

½ (5 x — 2) = 2 x + 4.

Решение . Это уравнение с дробью. Очистить дроби путем умножения обеих сторон на 2:

5 x -2 = 4 x + 8
5 x — 4 x = 8 + 2
x = 10.

В следующих задачах очистить дроби и решить для x :

Чтобы увидеть каждый ответ, наведите указатель мыши на цветную область.
Чтобы закрыть ответ еще раз, нажмите «Обновить» («Reload»).
Сначала решите проблему сами!

Задача 1. x
2
x
5
= 3
LCM — это
10.Вот очищенное уравнение и его решение:
5 х 2 x = 30
3 х = 30
х = 10.

При решении любого уравнения с дробями в следующей строке вы пишете —

5 x — 2 x = 30

— должно иметь без дробей .

Задача 2. x
6
= 1
12
+ x
8
LCM — это
24.Вот очищенное уравнение и его решение:
4 х = 2 + 3 х
4 x — 3 x = 2
х = 2
Проблема 3. x — 2
5
+ x
3
= x
2
LCM — это
30. Вот очищенное уравнение и его решение:
6 (x -2) + 10 x = 15 х
6 x — 12 + 10 x = 15 х
16 x -15 x = 12
х = 12.

Задача 4. Дробь равна дроби.

x — 1
4
= x
7
LCM — это
28. Вот очищенное уравнение и его решение:
7 ( x — 1) = 4 х
7 x — 7 = 4 х
7 x — 4 x = 7
3 х = 7
х = 7
3

Мы видим, что когда единственная дробь равна единственной дроби, тогда уравнение может быть очищено «перекрестным умножением».«

Если
а
б
= c
d
,
, затем
объявление = до н.э. .
Задача 5. x — 3
3
= x — 5
2
Вот очищенное уравнение и его решение:
2 ( x — 3) = 3 ( x -5)
2 x — 6 = 3 х -15
2 x — 3 x = — 15 + 6
х = −9
х = 9
Проблема 6. x — 3
x — 1
= x + 1
x + 2
Вот очищенное уравнение и его решение:
( x — 3) ( x + 2) = ( x — 1) ( x + 1)
x ² — x — 6 = x ² — 1
х = −1 + 6
х = 5
х = −5.
Задача 7. 2 x — 3
9
+ x + 1
2
= x — 4
LCM — это
18. Вот очищенное уравнение и его решение:
4 x — 6 + 9 x + 9 = 18 x — 72
13 х + 3 = 18 x — 72
13 x — 18 x = — 72 — 3
−5 х = −75
х = 15.
Задача 8. 2
x
3
8 x
= 1
4
LCM — это
8 х . Вот очищенное уравнение и его решение:
16–3 = 2 х
2 х = 13
х = 13
2

2-й уровень

Следующий урок: задачи Word

Содержание | Дом


Сделайте пожертвование, чтобы TheMathPage оставалась в сети.

Добавить комментарий

Ваш адрес email не будет опубликован. Обязательные поля помечены *